You are on page 1of 80

Representation of Groups

UNIT 7 REPRESENTATION OF GROUPS

Structure Page
7.1 Introduction 5
Objectives
7.2 Examples 5
7.3 Complete Reducibility and Maschke’s Theorem 10
7.4 Summary 14
7.5 Solutions/Answers 14

7.1 INTRODUCTION

The representation theory of finite groups has its origins in the correspondence
that took place between two mathematicians, R. Dedekind and F.G. Frobenius
in the year 1898. This theory helps us understand theory of finite groups by
translating problems related to finite groups to problems related to a particular
family of finite groups, namely, finite subgroups of GLn (C) . Once we make Fig.1: F. G. Frobenius
(1849-1917)
this translation, we can apply tools and techniques from both Linear Algebra
and from the theory of finite groups to solve the problems. This approach has
proved very successful and has been useful in proving deep theorems in Group
Theory which would have been impossible or difficult to prove otherwise. To
give an example, representation theory has contributed to the solution of the
classification problem of finite simple groups.

Further, several fundamental results of representation theory have major


applications in Theoretical Physics, Chemistry, Computer Vision,
Cryptography, and many areas within mathematics itself like Number Theory.

In this unit, we shall help you study some basics about representations of finite
groups, irreducible representations and Maschke’s theorem on complete
reducibility of representations over C .

Objectives

After studying this unit, with Sections 1, 2, 4 of Chapter 9 of the textbook,


you should be able to:

• define, and give examples of, group representations;


• describe G-invariant forms;
• explain why every matrix representation of a finite group is conjugate to a
unitary representation;
• prove, and use, Maschke’s theorem.

7.2 EXAMPLES

Read Section 1 of Chapter 9 in the textbook, up to Equation 1.6. Then read


the following.

5
Study Guide-II Remark: Regarding Page 308, Lines 5 and 6, starting with `It is also easy to
write down…’, the case of D n is explained further on Page 315, fourth
paragraph, starting with `For example, consider …’.

Let now look at some examples of matrix representations.

Example 1: Given any group G and vector space V, define


ρ : G → GL(V ) by ρ(g) = I for all g ∈ G. Show that ρ is a representation of G.
(This is called the trivial representation.)

Solution: Since ρ (gh) = I = ρ (g) ρ(h) ∀g, h ∈ G, ρ is a homomorphism, and


hence, a representation of G.
***

In order to check that a map φ : G → GL n (C) is a group homomorphism for a


group G of order m we have to check that φ(ab) = φ(a)φ(b) for m(m − 1) pairs
a, b. If we want to check that a map φ : D 5 → GL n (C) is a group
homomorphism, we will have to check that φ(ab) = φ(a)φ(b) for 10 × 9 = 90
pairs, which is a very tedious task. However, the results that we have studied
in Unit 4 come to our rescue.

Suppose G = x1 , x 2 ,K , x m ; r1 , r2 , K , rk is a group. Recall that, this means that


there is unique onto group homomorphism π : F → G where F is the free group
on the set S = {x1 , x 2 ,K , x m } . Further, the kernel of π is the smallest normal
subgroup of F containing the elements {r1 , r2 ,K , rk } . Let us denote the kernel
of π by N.

Let G ′ is another group and φ : S → G′ be a map. Suppose r ∈ {r1 , r2 ,K , rk }


and we write r = x ij11 x ij22 L x ijnn where i1 , i 2 , K, i m ∈{1, 2, K, m} and each ji is 1

( ) φ(x ) ( )
j1 j2 jn
or −1 . We say φ respects r, if φ x i1 i2 L φ xin = 1 in G ′ .

Proposition 1: Let G = x1 , x 2 ,K x m ; r1 , r2 ,K , rk and φ : S → G′ is a map that


respects all the relations r1 , r2 ,K , rk . Then, there is a unique group
homomorphism ψ : G → G ′ such that φ(x i ) = ψ (π(x i )).
)
Proof: The map φ extends to a group homomorphism φ from F to G' by the
mapping property of free groups. (See Proposition (8.1) page 202 of Artin.)
)
{
Note further that, φ maps any word in S1 = x1 , x1−1 , x 2 , x 2−1 ,K , x m , x m −1 to }
{ −1
the corresponding word in S = φ(x1 ), φ(x1 ) ,K , φ(x m ), φ(x m )
'
1
−1
}. In
particular, if r = x ij11 x ij22 K x ijnn and r ∈ {r1 , r2 ,K , rk } ,
) ) j1 ) ) )
( ) ( ) ( ) ()
j2 jn
φ(r) = φ x i1 φ x i2 L φ x i n . Let us write Ker φ = N′. For each
r ∈ {r1 , r2 , K , rk } , we have

6
) ) ) )
( ) φ(x ) ( )
j1 j2 jn Representation of Groups
φ(r) = φ x i1 i2 L φ xin
)
= φ( x ) φ(x ) Lφ ( x )
j1 j2 jn
i1 i2 in (Q φ is an extension of φ)
= 1(Q φ respects r )
)
It follows that ri ,1 ≤ i ≤ k, are in N′. So, N', the kernel of φ, contains N, the
kernel of π . We have the following situation:
)
φ
F G′

π
G
Fig. 2

Now, we apply proposition (8.3) in page 221 to obtain a group homomorphism


)
ψ : G → G ′ such that ψ ( π ( x i ) ) = φ ( x i ) = φ ( x i ) . (See Figure below.)
)
φ
F G′

π ψ
G
Fig. 3

Example 2: Consider the Klein 4-group given by
V4 = x, y; xyxy, x 2 , y 2 .
Obtain a non-trivial representation of V4 .

Solution: V4 = {1, x, y, xy}. Let F be a field, Char(F) ≠ 2. Define


ρ : {x , y} → GL 2 (C)
by
⎡ −1 0⎤ ⎡1 0⎤
ρ(x) = ⎢ ⎥ = a, ρ(y) = ⎢ = b.
⎣ 0 1⎦ ⎣0 −1⎥⎦
You can easily check that
⎡1 0 ⎤ ⎡1 0⎤
a 2 = b2 = ⎢ ⎥ , abab = ⎢ .
⎣0 1 ⎦ ⎣0 1 ⎥⎦

Therefore, ρ respects the relations xyxy and x 2 and y 2 . So, by proposition 1,


ρ extends to a representation of V4 such that
⎡ −1 0 ⎤
ρ(xy) = ⎢ ⎥.
⎣ 0 −1⎦
Here, we are abusing the notation and using ρ to represent the extension of ρ
to V4 also.  The dihedral group of order
2nis Dn = x,y ;xn,y2,xyxy
Example 3: Consider the dihedral group of order 8, D4. Obtain a non-trivial In some books this is denoted
representation of D4. by D 2n .

7
studying characters later in the book and this block. For now, let us look at Representation of Groups
some examples.

Example 6: Show that the representations ρ and ρ′, defined in Example 5, are
irreducible.

Solutions: Since ρ and ρ′ are conjugate representations, ρ is irreducible if and


only if ρ′ is irreducible. Let us show that ρ is irreducible.
⎧⎪ ⎡α⎤ ⎫⎪
Here the representation space is C 2 = ⎨ ⎢ ⎥ α, β ∈ C⎬ .
⎪⎩ ⎣β ⎦ ⎪⎭
⎡α ⎤
Let W be a non-zero D6 - invariant subspace of V . Then ∃x = ⎢ ⎥ ∈ W, x ≠ 0 .
β ⎣ ⎦
⎡β ⎤
Since W is D6 - invariant, Yx = ⎢ ⎥ ∈ W, where Y is as in Example 5.
⎣α ⎦
⎡α + β ⎤ ⎡ α −β ⎤
So, x + Yx = ⎢ ⎥ ∈ W and x − Yx = ⎢ ⎥ ∈ W. Therefore, either
⎣α + β ⎦ ⎣− ( α − β ) ⎦
α + β ≠ 0 or α − β ≠ 0, since x ≠ 0 .
⎡1⎤ ⎡ 1⎤
Since at least one of ( α + β ) , ( α − β ) is non-zero, either ⎢ ⎥ ∈ W or ⎢ ⎥ ∈ W .
⎣1⎦ ⎣ −1⎦
⎡ 2π i ⎤
⎡1⎤ ⎡1⎤ ⎢ e 3 ⎥ ⎡1⎤ ⎡1⎤
Suppose ⎢ ⎥ ∈ W. Then X ⎢ ⎥ = ∈ W. Also, ⎢ ⎥ and X ⎢ ⎥ are linearly
⎣1⎦ ⎢e −2 π 3 ⎥
i
⎣1⎦ ⎣1⎦ ⎣1⎦
⎣ ⎦
independent.(Why ?). So dim C W = 2 and hence W = V .
⎡1⎤
Similarly, you can show that if ⎢ ⎥ ∈ W, W = V.
⎣ −1⎦
Thus, the only D6 -invariant subspaces of V under ρ are (0) and V . Thus, ρ
is irreducible.
***

Example 7: Let G = D 4 = x, y; x 4 , y 2 , yxyx , and let ρ be as defined in


Example 3. Show that ρ is irreducible.

Solution: Suppose W is a non-zero D 4 -invariant subspace of V under ρ . Let


⎡α ⎤
ρ (x) = A, ρ (y) = B. Let v = ⎢ ⎥ ∈ W be non-zero. Then
⎣β ⎦
⎡ 0 1 ⎤ ⎡α ⎤ ⎡ β ⎤ ⎡1 0 ⎤ ⎡α ⎤ ⎡ α ⎤
Av = ⎢ ⎥ ⎢ ⎥ = ⎢ ⎥ ∈ W and Bv = ⎢ ⎥ ⎢ ⎥ = ⎢ ⎥∈W .
⎣ −1 0 ⎦ ⎣β ⎦ ⎣ −α ⎦ ⎣0 −1⎦ ⎣β ⎦ ⎣−β⎦
⎡ α +β ⎤ ⎡1⎤
So, Av + Bv = ⎢ ⎥ = (α + β) ⎢ ⎥ ∈ W and
⎣−(α + β) ⎦ ⎣−1⎦
⎡α − β ⎤ ⎡1⎤
Bv − Av = ⎢ ⎥ = (α − β) ⎢ ⎥ ∈ W .
⎣α − β ⎦ ⎣1⎦
⎡ 1⎤ ⎡1⎤
Now, either α + β ≠ 0 or α − β ≠ 0, since v ≠ 0. So, either ⎢ ⎥ ∈ W or ⎢ ⎥ ∈ W .
⎣ −1⎦ ⎣1⎦

11
Study Guide-II ⎡1⎤ ⎡1⎤ ⎡1 ⎤
If ⎢ ⎥ ∈ W, then B⎢ ⎥ = ⎢ ⎥ ∈ W , so that dim C W = 2 , and hence W = V .
⎣1⎦ ⎣1⎦ ⎣− 1⎦
⎡1⎤ ⎡1⎤
Similarly, if ⎢ ⎥ ∈ W, so does ⎢ ⎥ ∈ W, and W = V.
⎣ −1⎦ ⎣1⎦
Therefore, ρ is irreducible.
***

Example 8: Let C3 = x | x 3 = 1 = {1, x, x 2} . Consider the representation ρ of


C3 on V = C3 given by
⎡1 0 0⎤ ⎡0 0 1 ⎤
ρ : C 3 → GL 3 (C) : 1 → I = ⎢0 1 0⎥ , x → X = ⎢⎢1 0 0⎥⎥ .
⎢ ⎥
⎢⎣0 0 1⎥⎦ ⎢⎣0 1 0⎥⎦
Note that X maps e1 to e 2 , e 2 to e3 and e3 to e1.
Show that ρ is reducible. What does Maschke’s theorem say in this case?

⎡1⎤ ⎡1⎤ ⎡1⎤ ⎡1⎤ ⎡1⎤


Solution: Let W = 〈 〉
⎢1⎥ . Then x ⋅ ⎢1⎥ = ρ x ⎢1⎥ = X ⎢1⎥ = ⎢1⎥ ∈ W. So, W is a
⎢⎥ ⎢⎥ ( )⎢ ⎥ ⎢ ⎥ ⎢ ⎥
⎢⎣1⎥⎦ ⎢⎣1⎥⎦ ⎢⎣1⎥⎦ ⎢⎣1⎥⎦ ⎢⎣1⎥⎦
C3 -invariant subspace under ρ, and W ≠ C 3 . Hence, ρ is not irreducible.
Here, ρ is unitary. So, the usual inner product on C 3 is G -invariant.
Also, let
⎧ ⎡1⎤ ⎫
⊥ ⎪ ⎢ ⎥ ⎪
W = ⎨v ∈ C v, ⎢1⎥ = 0⎬
3

⎪ ⎢⎣1⎥⎦ ⎪
⎩ ⎭
be the orthogonal complement of W under the standard inner product on C 3 .
Now
⎡1⎤ ⎡α ⎤
⎢ ⎥
v, ⎢1⎥ = α + β + γ , where v = ⎢⎢β ⎥⎥ .
⎢⎣1⎥⎦ ⎢⎣ γ ⎥⎦
So,
W ⊥ = {(α, β, γ) α + β + γ = 0}.
Check that,
⎡1 ⎤ ⎡0 ⎤
W is generated by ⎢ − 1⎥ and ⎢⎢1 ⎥⎥ .
⊥ ⎢ ⎥
⎢⎣0 ⎥⎦ ⎢⎣ −1⎥⎦

So, V = W ⊕ W ⊥ gives the decomposition of V as a direct sum of ρ - invariant


subspaces. Then ρ = ρ1 ⊕ ρ2 is a direct sum of non-trivial representations,
ρ denotes the restriction where ρ = ρ1 and ρ = ρ2 . We leave it to you as an exercise to check that
w w w⊥
of ρ to W.
W ⊥ is also reducible.
***
12
Example 9: Consider S3 = (1 2 3) = σ, (1 2) = τ = {1, σ, σ 2 , τ, σ τ, σ 2 τ} , and
Representation of Groups

ρ : {σ, τ} → GL3 (C) defined by


⎡0 0 1 ⎤ ⎡0 1 0 ⎤
σ → S = ⎢1 0 0 ⎥ , τ → T = ⎢⎢1 0 0⎥⎥ .
⎢ ⎥
⎢⎣0 1 0 ⎥⎦ ⎢⎣0 0 1 ⎥⎦
Note that, S3 ≅ D3 = x, y; x 3 , y 2 , yxyx under the map σ a x, τ a y because
σ3 = τ2 , τστσ = 1. Also, S3 = I, T 2 = I, TSTS=I . You can easily check that
ρ extends to a group representation ρ : S3 → GL 2 (C) such that
ρ(σ ) = S, ρ(τ)=T. Write ρ as a sum of irreducible representations.

⎡1⎤ ⎧ ⎡α ⎤ ⎫
⎢1⎥ ⎪⎢ ⎥ ⎪
Solution: Let W = = e1 + e 2 + e 3 = ⎨ ⎢α ⎥ α ∈ C ⎬
⎢⎥
⎢⎣1⎥⎦ ⎪ ⎢α ⎥ ⎪
⎩⎣ ⎦ ⎭
where Β = {e1 , e 2 , e3 } is the standard basis of C 3 . Note that, σ and τ act on the
basis B as follows:
σ ⋅ e1 = e 2 , σ ⋅ e 2 = e 3 , σ ⋅ e 3 = e1 , τ ⋅ e1 = e 2 , τ ⋅ e 2 = e1 , τ ⋅ e 3 = e 3 .

So, W is an S3 -invariant subspace of V under ρ .


Also, let
⎡ 1 ⎤ ⎡ 0⎤

W = e1 − e2 , e 2 − e3 = 〈 ⎢ ⎥ ⎢ ⎥ 〉
⎢ −1⎥ , ⎢ 1 ⎥ .
⎢⎣ 0 ⎥⎦ ⎢⎣−1⎥⎦
As before, since the representation ρ is unitary, the standard inner product on
C 3 is ρ invariant. So, W ⊥ is ρ -invariant subspace of C 3 and V = W ⊕ W ⊥ .
So, ρ = ρ1 ⊕ ρ 2 , where ρ1 = ρ W and ρ2 = ρ W⊥
. Since W is one dimensional,
ρ1 is one dimensional.

Note that if ρ2 is reducible, W ⊥ will have a one dimensional subspace W1


which is invariant under S and T. If u ∈ W1 , u ≠ 0 , then Su = λu, Tu = μu.
Further, from S3 = I, it follows that S3u = λ 3 u = u. So, λ 3 − 1 = 0 and
{ }
therefore λ ∈ 1, ω, ω2 , where ω is a primitive cube root of unity. Similarly,
from T 2 = I, it follows that μ 2 = 1 and μ ∈ {1, −1}. Again, from TSTS = I, if
follows that (λμ)2 = 1. This can happen only if λ = 1.

In this case, we have Su = u. Let us write u = α(e1 − e2 ) + β(e 2 − e3 ). Note


that, Se1 = e 2 , Se 2 = e3 and Se3 = e1. So,
Su = α (e 2 − e3 ) + β(e3 − e1 )
= α(e 2 − e3 ) − β((e1 − e 2 ) + (e 2 − e3 ))
= −β(e1 − e 2 ) + (α − β)(e 2 − e3 )
= u = α(e1 − e2 ) + β(e 2 − e3 )
Since e1 − e 2 and e 2 − e3 are linearly independent, it follows that α = −β,

13
Study Guide-II β = α − β. Solving, we get α = 0, β = 0. But, this contradicts our assumption
that u ≠ 0. So, ρ2 is irreducible.
***

Try the following exercises now.

E1) Let ρ : G → GL 2 (C) be a representation of a finite group G . If there


EXERCISES
exist elements x, y ∈ G such that ρ(x)ρ(y) ≠ ρ(y)ρ(x) , show that ρ is
irreducible.

E2) Let G be a group of order n, which acts on a set X = {x1 ,K, x m }. Let V
be an m-dimensional vector space with basis B = {e xi i = 1 ,K, m}. Define
ρ : G → GL (V) by ρ (g) (Σ αi e xi ) = Σ αi egxi . Show that ρ is a representation
i i
of G. (This is called the permutation representation associated with
X.)

E3) Consider W ⊥ = e1 − e 2 , e 2 − e3 in example 8.


In this exercise, we will show that W ⊥ is reducible. Let ωbe a primitive
cube root of unity. Let
u = (1 − ω)e1 + (ω2 − 1)e 2 + (ω − ω2 )e3 .
Check that u ∈ W ⊥ and Xu ∈ W ⊥ . Why is this enough to show that W ⊥ is
reducible?

EXERCISES Further, try Exercises 1, 2, 4, 5 under Section 4, P. 337 of the textbook. In Q5


assume that G is finite.

We now come to the end of this introductory discussion on representations of


finite groups. We shall summarise what we have covered in it.

7.4 SUMMARY

In this unit we have covered the following points:

1) The definition, and examples, of a group representation.

2) a) We have proved that every finite subgroup of GL n is conjugate to a


subgroup of U n .
b) Therefore, if ρ is a representation of a finite group G, then ρg is
diagonalisable ∀g ∈ G.

3) The definition, and examples, of an irreducible representation.

4) The proof, and applications, of Maschke’s theorem, which says that every
representation of a finite group is a direct sum of irreducible
representations.

14
Representation of Groups
7.5 SOLUTIONS/ANSWERS

Solutions to Exercises 1, 2, 4, 7, 8 and 9, in the section ‘Definition of a


Group Representation’, Section 1, Page 335.

Q1) Let ρ : G → GLn ( F ) , F is a field.


Define det ρ : G → GL1 (F) : (det ρ) (g) = det ( ρ (g) )
Now, det ρ ( g1 .g 2 ) = det ( ρ ( g1 .g 2 ) )
= det ( ρ ( g1 ) ρ ( g 2 ) )
= det ( ρ ( g1 ) ) det ( ρ ( g 2 ) )
= det ρ ( g1 ) det ρ ( g 2 )

This shows that det ρ is a one-dimensional representation.

Q2) Let ρ : G → GLn ( F ) be a faithful representation such that ρ ( g ) is diagonal


for every g ∈ G . Let g1 ,g 2 ∈ G . Then ρ ( g1 ) ρ ( g 2 ) = ρ ( g 2 ) ρ ( g1 ) , since
diagonal matrices commute. ⇒ ρ ( g1g 2 ) = ρ ( g 2g1 ) .
⇒ g1g 2 = g 2 g1 , sin ce ρ is faithful.
This shows that G is abelian.

Q4) Let ρ : S5 → GL1 ( F ) = F* be a representation and let (i j) be any


transposition. Let α = ρ ( i j) ∈ F* . Then
1 = ρ (1) = ρ ( i j) = ( ρ ( i j) ) = α 2 ,
2 2

so that α = ρ ( i j) = 1or − 1 if char ( F ) ≠ 2, and


α = ρ ( i j) = 1 = −1 if char ( F ) = 2 .

Case-1 ( ρ ( T ) = 1 for some transposition T ∈ S5 ): By Proposition 6.10 (c),


Chapter 6, of the textbook, any two transpositions are conjugate.
Also, ( F* ,.) is an abelian group. Therefore, in this case ρ ( T ) = 1 for all
transpositions.

Further, as any permutation is a finite product of transpositions, we get


ρ ( g ) = 1 for all g ∈ S5 .

Case-2 ( ρ ( T ) = −1 for some transposition T ∈ S5 ):


As above, ρ ( T ) = −1 for all transpositions T.
⎧ 1 if g is even
Then for any g ∈ S5 , ρ ( g ) = ⎨ = sign ( g )
⎩−1 if g is odd

Thus, ρ is the sign representation.

Remark: Q4, and the solution above, remains the same if S5 is replaced
by Sn .

15
N ⊆ Ker ρ′ , then ρ : G N → GL ( V ) , defined by ρ ( gN ) = ρ′ ( g ) , is a Representation of Groups

representation of G N .
-----------------------------------------------------------------------------------------------

Solutions to Exercises 1, 2, 3, 4, 6, 7 of Section 2, Page 336.

Q1) a) X, Y = X* BY
Let C3 = {1, g, g 2 } , with g 3 = 1. Then
ρ : C 3 → GL 2 (C) : ρ(1) = 1, ρ(g ) = A , and extend it to form a
homomorphism. Here A 3 = I.
Then {ρ (g)X, ρ (g)Y} = {AX,AY} = ( AX ) BAY
*

= X* A* BAY = X* BY, since A* BA = B


= X, Y
Similarly, you can check that
{ρ (g 2
} {
)X , ρ (g 2 )Y = A 2 X , A 2 Y }
= X, Y
Hence, the form X, Y = X*BY is G-invariant.

b) Applying the Gram-Schmidt process of orthonormalisation to


⎪⎧ ⎡1 ⎤ ⎡ 0⎤ ⎪⎫
⎨e1 = ⎢ ⎥ ,e 2 = ⎢ ⎥ ⎬ in C with respect to the form X,Y = X BY,
2 *

⎩⎪ ⎣ ⎦
0 ⎣ ⎦ ⎭⎪
1
2 2
we see that e1 = and e1 , e 2 = . So, an orthonormal basis is
3 3
⎧⎪ 3 ⎡1 ⎤ ⎡ − 1 ⎤ ⎫⎪
⎨e1′ = ⎢0⎥ , e ′2 = ⎢
2 ⎥ ⎬ for this new form.
⎪⎩ 2 ⎣ ⎦ ⎢⎣1 ⎥⎦ ⎪⎭
⎡ 3 −1 ⎤
The change of basis matrix P is ⎢ 2 2⎥
⎢⎣ 0 1 ⎥⎦
⎡ −1 − 3 ⎤
So P AP = ⎢⎢ 2
−1 2 ⎥.
3 −1 ⎥
⎢⎣ 2 2 ⎥⎦

Q2) Do this on the same lines as the proof of Th. 2.2, based on Th. 2.6,
Chapter 9.

Q3) Let ρ : G → SL 2 (R ) be a faithful irreducible representation. By Q2, there


is an invertible matrix P such that Pρ (g) P −1 ∈ O2 ∀g ∈ G. Since ρ(g) has
determinant 1, Pρ(g)P −1 also has determinant 1. So, Pρ(g)P −1 ∈ SO2 .
From Theorem 5.5, page 124 of Artin, we know that
⎧⎪ ⎛ cos θ − sin θ ⎞ ⎫⎪
SO 2 (R) = ⎨ ⎜⎜ ⎟⎟ θ ∈ R ⎬ = K (say).
⎪⎩ ⎝ sin θ cos θ ⎠ ⎪⎭
But
K ~ {cos θ + i sin θ | θ ∈ R} = {z ∈ C* | | z | = 1} = S1 (say)
under the map φ defined by

17
Study Guide-II ⎛ cos θ − sin θ ⎞
⎜ ⎟ a cos θ + i sin θ.
⎝ sin θ cos θ ⎠
( )
So, G′ = φ Pρ(g)P −1 ⊆ S1.
But, any finite subgroup of S1 is cyclic. To see this, note that, every
element of a group of order n contained in S1 will satisfy x n − 1 = 0 . So,
any finite group of order n is contained in
2 πi

{ 1 n
}
C n = z ∈ S z − 1 = 0 = μ n where μ n = e n
.
Since C n is cyclic and any subgroup of a cyclic group is cyclic, it follows
that any finite subgroup of S1 is cyclic.
So, G ′ is also cyclic.
Since G ′ is an isomorphic copy of G, G is also cyclic.

Q4) As in question 3, we can show that there is an invertible matrix P such


that Pρ(g)P −1 ∈ O 2 for all g ∈ G. From Theorem 3.4, page 164 of Artin, it
follows that G is either cyclic or dihedral.

Q6) You can check that the unitary operators form a subgroup.
Now suppose ρ (g) ∈ U (V)∀g ∈ G.
Then ρ (g)v, ρ (g)w = v, w ∀v, w ∈ V
Conversely, let < , > be G-invariant.
Then ρ (g)v, ρ (g)w = v, w ∀g ∈ G, so that ρ (g) ∈ U (V).

Q7) Since the form is non-degenerate, x, y = 0∀ y ∈ V ⇒ x = 0.


Since the form is skew-symmetric, x, y = − y, x , so that
x, x = 0∀ x ∈ V.

1
a) Define {v, w} = Σ ρ (g) v, ρ (g) w
G g∈G
Then you can check that { , } will be G -invariant & skew-
symmetric.
b) By Ch. 7, Th. 8.5 of the book, there is a basis B of V such that the
⎡0 I⎤
matrix of the form with respect to B is J 2n = ⎢ ⎥.
⎣ -I 0⎦
Then, as in the proof of Th. 2.2, Chapter 9, any finite subgroup of
GL (V), where V = C 2n lies in the symplectic group SP2 n (C) .
-----------------------------------------------------------------------------------------------

Solutions to E1, E2 and E3 of this unit, and Exercises 1, 2, 4, 5 of Section 4,


Page 337.

E1) Suppose ρ is not irreducible. Then, by Maschke’s theorem, ρ is a direct


sum of two 1- dimensional representations (which, of course, are
irreducible). But then, C 2 = W1 ⊕ W2 , W1 = C . So, with respect to a
suitably ordered basis B of C 2 , the matrices of ρ(x) and ρ(y) are
⎡α 0⎤ ⎡ γ 0⎤
diagonal that is, [ρ( x )]B = ⎢ ⎥ , [ρ( y)]B = ⎢ ⎥ , where
18 ⎣ 0 β⎦ ⎣0 δ ⎦
α, β, γ, δ ∈ C . So, ρ (x) ρ (y) = ρ (y) ρ (x) , a contradiction. Hence ρ Representation of Groups
must be irreducible.

E2) Since G acts on X, 1.x = x, g1 (g 2 .x) = (g1 g 2 ) x.


∴ρ is a linear representation of G.

E3) We have u = (1 − ω)(e1 − e 2 ) + (ω2 − ω)(e2 − e3 ). So, u ∈ W ⊥ .


We have Xe1 = e 2 , Xe 2 = e3 , Xe3 = e1 . So,
Xu = (1 − ω)e2 + (ω2 − 1)e3 + (ω − ω2 )e1
= ωu

So, W is reducible. Since X generates the image of C3 in GL3 (C) , it
follows that gu is a scalar multiple of u for each element g of C3 .

⎡a ⎤
Q1) Let W be a G - invariant subspace of V and v = ⎢⎢ b ⎥⎥ ≠ 0 be in W.
⎢⎣ c ⎥⎦
⎡a ⎤ ⎡ a ⎤ ⎡ −a ⎤ ⎡ c ⎤ ⎡ −a ⎤
Then R y1 ⎢ b ⎥ = ⎢ −b ⎥ ∈ W. Similarly, ⎢⎢ − b ⎥⎥ , ⎢⎢ a ⎥⎥ , ⎢⎢ b ⎥⎥ ∈ W.
⎢ ⎥ ⎢ ⎥
⎢⎣ c ⎥⎦ ⎢⎣ −c ⎥⎦ ⎢⎣ c ⎥⎦ ⎢⎣ b ⎥⎦ ⎢⎣ −c ⎥⎦
⎡1 ⎤ ⎡0⎤ ⎡0⎤
So, ⎢⎢0 ⎥⎥ , ⎢⎢0 ⎥⎥ , ⎢⎢1 ⎥⎥ ∈ W. ∴W = V
⎢⎣ 0 ⎥⎦ ⎢⎣1 ⎥⎦ ⎢⎣ 0⎥⎦
∴ R is irreducible.

Q2) Let ρ : C n → GL m (C), m > 1 , be irreducible. Let Cn = g . Since


m > 1, ρ (g) has an eigenvalue λ , and an eigenvector X ∈ C m . Then
ρ (g)X = λX. Thus, X is Cn -invariant, making ρ reducible. This is a
contradiction. Therefore, ρ can be irreducible only if m = 1, that is,
dimension ρ = 1.

1
Q4) a) Let v = Σ ( ρ (g) v ) . Then, by reasoning as in the proof of
G g∈G
Lemma 2.8, Chapter 9, g.v = v ∀g ∈ G.

b) If ρ is irreducible, then dim V = 1 with V = < v > , because otherwise,


V would have < v > as a direct summand.

Q5) w = Σ ρ (h) v
h∈H
Then ρ(g)w = Σ ρ(gh) v
h∈H

ρ(g)w = ρ(g′)w ⇒ gg −1 fixes w ⇒ gg −1 ∈ H


∴ Gw = G : H

19
Study Guide-II
UNIT 8 CHARACTERS

Structure Page
8.1 Introduction 20
Objectives
8.2 Examples 20
8.3 Schur’s Lemma 24
8.4 Summary 25
8.5 Solutions/Answers to Exercises 26

8.1 INTRODUCTION

In this unit, we shall focus on characters associated with finite-dimensional


representations of a finite group over the field  of complex numbers.
Character theory plays a vital role in understanding the structure of finite
groups and the classification of finite simple groups. Here you will study part
of Chapter 9 of the textbook, which looks at characters of finite groups,
orthogonality relations and their applications. We will also discuss basic
results like Schur’s Lemma, and explicitly determine character tables of cyclic
groups and some permutation groups.

Objectives

After studying Sections 5, 6, 8 and 9 of Chapter 9 of ‘Algebra’ by Artin, and


this unit, you should be able to:
• define, and give examples, of a character of a representation;
• prove, and apply, some basic properties of a character;
• obtain all the irreducible representations of some finite groups;
• prove, and use, the orthogonality representations satisfied by irreducible
characters of a group;
• prove Schur’s Lemma, and apply it for proving some properties of
irreducible representations.

8.2 EXAMPLES

Read Sections 5 and 6, Chapter 9, Pages 316 - 321 of the textbook, along
with the notes below.

1) ρ : G → GL (V) and ρ′ : G → GL (V′) are isomorphic (or equivalent) iff a


NOTE
vector space isomorphism T : V → V′ exists such that the following
diagram is commutative for each g ∈ G

T
V ∼ V′
↓ → ↓
ρg ↑ T
∼ ↑ ρg′
V → V′

i.e., T o ρg = ρ′g o T .

20
Characters
2) χ : G →  is well-defined because if B1 and B2 are two ordered bases of V,
where ρ : G → GL (V), then by (1) above ⎡⎣ρg ⎤⎦ B and ⎡⎣ρg ⎤⎦ B′ are similar
matrices. Therefore, their traces are the same. Thus, the definition of the
character χ of ρ is independent of the ordered basis chosen.

3) In the development of Character Theory of Finite Groups and its


applications, a fundamental result is Theorem 5.9, Page 318 of the
textbook.

Definition: Let r be the number of distinct conjugacy classes in a finite group


G, and let χ1 , χ 2 , L , χ r be the irreducible characters of G . Let g1 , K , g r be
representatives of the conjugacy classes C1 , C2 ,L, Cr of G . The r × r matrix
( χ ( g )) ,1 ≤ i, j ≤ r , is called the character table of G , and is given as below.
i j

Conjugacy classes
C1 C2 L Cr ← order of the conjugacy class
g1 g2 L gr ← representative elements

χ1 χ1 ( g1 ) χ1 ( g 2 ) L χ1 ( g r )
χ2 χ 2 (g1 ) χ2 (g 2 ) L χ 2 ( g r )
irreducible
characters M M M L M
χr χ r (g1 ) χr ( g 2 ) L χ r ( g r )

Remark: The r × r matrix ⎡⎣χi ( g j )⎤⎦ is invertible since χ1 ,K , χ r form a basis of


the vector space of all class functions of the group G.

In the textbook you will find the character tables of the symmetric group
S3 (isomorphic to D3 ), the cyclic group of order 3 and the tetrahedral group.
Let us look at some other examples.

Example 1: Write down the character table of the dihedral group


D 4 = x , y : x 4 , y 2 , yxyx .

Solution: The conjugacy classes of D 4 are


{1} ,{x, x −1 = x 3 } ,{x 2 } ,{y, x 2 y},{xy, x 3 y} . Since D 4 has 5 conjugacy classes, it
has 5 irreducible representations over  . Let these be ρ1 , ρ2 , ρ3 , ρ 4 , ρ5 , having
dimensions d1 ,d 2 ,d3 ,d 4 ,d5 . Then, by Theorem 5.9 (c),
8 = D 4 = d12 + d 22 + d 32 + d 24 + d 25 each d i ≥ 1 , and d i | 8 . …………………. (1)
Moreover, the trivial representation [given by ρ (g) = 1∀g ∈ G] has dimension 1.
Hence, re-ordering if necessary, we see that (1) is true only if
d1 = d 2 = d 3 = d 4 = 1 and d 5 = 2 .

We now explicitly define all the ρi .


i) ρ1 : D 4 → GL1 (  ) =  * is the trivial representation, i.e., ρ1 ( x ) = 1 , and
ρ1 ( y) = 1 .

21
Study Guide-II
ii) ρ 2 : D 4 → GL1 ( ) =  * : ρ 2 ( x ) = 1, ρ 2 ( y) = −1 .

iii) ρ 3 : D 4 → GL 1 ( ) =  * : ρ 3 ( x ) = −1, ρ 3 ( y) = 1

iv) ρ 4 : D 4 → GL1 (  ) =  * : ρ 4 ( X) = −1, ρ 4 ( y) = −1

⎡ 0 1⎤ ⎡1 0 ⎤
v) ρ5 : D 4 → GL 2 ( ) : ρ 5 ( x ) = ⎢ ⎥ , ρ 5 ( y) = ⎢ ⎥ . (This is the
⎣ − 1 0 ⎦ ⎣ 0 − 1⎦
representation ρ defined in Example 2, Unit 7.)

Now, ρ1 , ρ2 , ρ3 , ρ4 are one-dimensional, hence irreducible. ρ5 is two-


dimensional, and we will need to check whether it is irreducible.
1
( ) ( )
Now, χ5 , χ5 = ⎡1.χ5 (1)χ5 (1) + 2.χ5 ( x ) χ5 ( x ) + 1.χ5 x 2 χ5 x 2
8 ⎢⎣
+ 2.χ5 ( y ) χ5 ( y ) + 2.χ5 ( xy ) χ5 ( xy ) ⎤ , using 5.4(b).

1
= [ 4 + 0 + 4 + 0 + 0] = 1
8
By Corollary 5.14, P. 319, of the textbook, this proves that ρ5 is irreducible.

The character table of D 4 can be computed, as below.

1 2 1 2 2
2
1 x x y xy
χ1 1 1 1 1 1
χ2 1 1 1 −1 −1
χ3 1 −1 1 1 −1
χ4 1 −1 1 −1 1
χ5 2 0 −2 0 0
***

Example 2: Obtain the character table of the Klein 4 group.

Solution: K 4 = x × y , where x 2 = e = y 2 .
Since K 4 is abelian, all its representations are one-dimensional, and their
characters will be homomorphisms. So the table is

1 1 1 1
1 x y xy
χ1 1 1 1 1
χ 2 1 −1 1 −1
χ3 1 1 −1 −1
χ 4 1 −1 −1 1
***

From problem 5 on page 338 of Artin, we know that the matrix


⎡ | Cj | ⎤
⎢ χ i (g j ) ⎥
⎢⎣ N ⎥⎦

22
is a unitary matrix. Since the conjugate transpose of a unitary matrix is also Characters
unitary, it follows that the columns of the matrix are also orthonormal. Writing
out the orthonormality relations for the columns we get
k | Ci | | C j |

m =1 N
χ m (g i )χ m (g j ) = δij
k
N
or, ∑
m =1
χ m (g i ) χ m ( g j ) =
| Ci | | C j |
δij .

In particular, if i ≠ j
k

∑χ
m =1
m ( g i ) χ m (g j ) = 0

We will see some applications of this results now.

Example 3: Determine the last row of the following character table of a group
G of order 12 which has 4 conjugacy classes.

1 3 4 4
x1 x2 x3 x4
χ1 1 1 1 1
What you will get is the
χ2 1 1 ω ω2 table for A4.

χ3 1 1 ω2 ω
χ4
where ω is a primitive cube root of unity.

Solution: Firstly, χ 4 (x1 ) = dimension of ρ4 = 12 − (12 + 12 + 12 ) = 3, by (5.10),


Chapter 9, of the textbook.

Next, using the column orthogonality relation in the note above, we have
χ1 (x1 ) χ1 (x 2 ) + χ 2 (x1 ) χ 2 (x 2 ) + χ3 (x1 ) χ3 (x 2 ) + χ 4 (x1 ) χ4 (x 2 ) = 0
⇒ 3 + 3 χ4 (x 2 ) = 0 ⇒ χ 4 (x 2 ) = −1.
Similarly, we get χ 4 (x 3 ) = 0 and χ 4 (x 4 ) = 0, since 1 + ω + ω2 = 0.
Thus, the missing row is [3 −1 0 0].
***

Remark: This could also have been done using Theorem 5.9 (a), Chapter 9, of
the textbook. But, while writing the equation down, you must also include the
number of conjugates, i.e., Cxi . So, for example,
χ 4 , χ1 = 0 ⇒ χ 4 (x1 ) ⋅ 1 ⋅ Cx1 + χ4 (x 2 ) ⋅ 1 ⋅ Cx 2 + χ 4 (x 3 ) ⋅ 1 ⋅ C x3 + χ 4 (x 4 ) ⋅ 1 ⋅ C x 4 = 0
⇒ χ 4 (x1 ) + 3 χ4 (x 2 ) + 4χ 4 (x 3 ) + 4χ 4 (x 4 ) = 0
In this way χ 4 , χ 2 = 0 = χ 4 , χ3 give two other equations. The required values
of the row can be obtained from solving these three equations.

We will now discuss some theorems which will help us obtain characters of a
group G, if we know the characters of G N , where N Δ G.

Theorem 1: Let G be a finite group, N Δ G and χ% be a character of G N . Then

23
Study Guide-II we can define a character χ of G with χ (g) = χ% (Ng) ∀ g ∈ G with the dimension
of χ being the same as the dimension of χ% .

Proof: Let ρ% be a representation of G N with character χ% . From Q 9, P. 336


of the textbook, you know that given ρ% , we obtain the representation
ρ : G → GLn () by ρ (g) = ρ% (Ng).
Let χ be the character of ρ . Then χ (g) = Tr (ρg ) = Tr ( ρ% Ng ) = χ% (Ng) ∀ g ∈ G.
Also the dimension of χ = χ (1) = χ% (N) = dimension of χ% . ■
This leads us to the following definition.

Definition: Let G be a group, N Δ G , χ% a character of G N . Then the character


χ of G, given by χ (g) = χ% (Ng) ∀g ∈ G , is called the lift of χ% to G.

We now use Theorem 1 to state a useful result, the proof of which is left as an
exercise for you.

Theorem 2: Let G be a finite group and N Δ G . There is a one-to-one


correspondence between the set of characters of G N and the set of characters
of G corresponding to the representations ρ of G which satisfy N ⊆ Ker ρ .
Further, under this correspondence, the irreducible characters of G N
correspond to the irreducible characters of G which have N in the kernel of the
associated representation.

EXERCISES Proof: Do this as an exercise, using Theorem 1. ■

On Page 321, to obtain the irreducible representations of T, you see the use of
Theorem 2 for lifting the representations of T H , which is cyclic.

EXERCISES Try Questions 2-6, 12 of Section 5, P. 338 of the textbook.

Now read Section 8, Chapter 9, of the textbook.

EXERCISES Try Questions 1, 2, 4, 6 of Section 8, P. 341-342, of the textbook.

8.3 SCHUR’S LEMMA

Read Section 9, Chapter 9 of the textbook.

Let ρ : G → GL ( V ) and ρ′ : G → GL ( V′ ) be finite-dimensional irreducible


representations of a finite group G over the field  , and let T : V → V′ be a G-
invariant linear transformation. Then Schur’s Lemma tells us that

a) either T is an isomorphism or T = 0.
b) Further, if V′ = V and ρ′ = ρ , then T = λI for some scalar λ ∈ . In fact,
this scalar λ is an eigenvalue of T.

Remarks: 1) Part (a) of Schur’s Lemma can also be stated as follows:

24
Let ρ : G → GL(V ) be an irreducible complex representation of Characters

a finite group G .

Let
D = EndG (V ) = {T : V → V | T is a G - invariant linear transformation}.
Then D is a division ring, which is finite-dimensional over the
field  .

2) Theorem 9.6 (b) crucially depends on the fact that  is an


algebraically closed field. It is not true for arbitrary fields, since
the proof uses the existence of eigenvalues, which may not
belong to the field if it is not algebraically closed.

3) If ρ : G → GL F ( V ) is a finite-dimensional irreducible
representation over a finite field F , then D = End G ( V ) is a finite-
dimensional division algebra over a finite field F . Thus D is a
finite division ring. But finite division rings are (commutative)
fields. Hence D = End G ( V ) is a finite field.

4) Orthogonality relations and their applications constitute the


major part of Character theory. These have already been
introduced in Section 2. By Maschke’s Theorem, every
representation is completely reducible, that is, it is a direct sum
of irreducible representations. Thus, if χ = χ ρ is the character of
a finite-dimensional complex representation ρ : G → GL ( V ) of a
r
finite group G , then χ = ∑ n i χi , where χ1 , χ 2 ,L, χr are the
i =1
irreducible characters of G and the ni s are non-negative
integers given by n i = χ, χi . Here we use the fact that
{χ1 , χ2 ,L, χr } is an orthonormal set.

With this we come to the end of this unit. Let us briefly review the points
covered in it.

8.4 SUMMARY

In this unit, and Sections 5, 6, 8, 9 of Chapter 9 of the textbook, we discussed


the following points:

1) The definition, and examples, of the character of a representation.

2) Some basic properties of a character.

3) The proof of orthogonality relations satisfied by the characters of


irreducible representations, and applications for finding entries of a
character table.

4) The proof of Schur’s Lemma, and some of its applications in the context
of irreducible representations.

25
Study Guide-II
8.5 SOLUTIONS/ANSWERS TO EXERCISES

Proof of Theorem 2: Let χ% be the character of the representation ρ% of G N .


Then, by Theorem 1, we get a representation ρ of G with character χ such that
χ (g) = χ% (Ng) ∀g ∈ G.
Now, from Q9, P.336, you know that if ρ is the representation of G
corresponding to χ , then N ⊆ Ker ρ.
Conversely, given a character χ corresponding to a representation ρ of G such
that Ker ρ ⊇ N , define ρ′ : G N → GLn () : ρ′ (Ng) = ρ (g).
You can check that ρ′ is a well-defined representation of G N , and its character
χ′ satisfies χ′ (Ng) = Tr (ρ′Ng ) = Tr (ρg ) = χ (g) ∀ g ∈ G.
1 2 1 2
Further, χ′ , χ′ = ∑ χ′ (Ng) = ∑ χ (g) . N , so that χ′ is
G G
N g∈G
G
N g∈ N

irreducible iff χ is irreducible.


Another way of seeing that χ′ is irreducible iff χ is irreducible is to note how
the matrices of ρ′ and ρ are related. This shows that ρ is irreducible iff ρ′ is
irreducible.

Solutions to Q 2, 3, 4, 5, 6, 12, Section 5, Page 338 of the textbook.

Q2) The 2-dimensional rotation representation is


⎡ 2π 2π ⎤
⎢c os n −sin n ⎥
ρ : Cn → GL2 () : ρ(x) = ⎢ ⎥ , where Cn = x .
⎢ sin 2π c os 2π ⎥
⎢⎣ n n ⎥⎦
As for C3 on P. 320 of the textbook, Cn has n irreducible representations
2 πi
2π 2π
ρ1 , K, ρn , each of dimension 1, where ξ = en = cos + i sin is a
n n
primitive n th root of unity given by ρi (x) = ξi .
If χ is the character of ρ , then
2πk
χ (x k ) = 2 cos = ξ k + ξ − k = ξ k + ξ(n −1) k for k ≥ 0.
n
This suggests that ρ = ρ1 ⊕ ρn −1. In fact, you can check that
−1 ⎡1 i ⎤ ⎡ξ k 0 ⎤
P A k P = Bk , where P = ⎢ ⎥ , A k = ρ (x ) , Bk = ⎢
k
−k

⎣ i 1⎦ ⎢⎣ 0 ξ ⎥⎦
∀ k = 0, 1, K , n − 1.
This shows that ρ = ρ1 ⊕ ρn −1 is the desired decomposition.

Q3) χ : G →  : χ (g) = tr (ρg ), where ρ is a representation of G.


Define ρ : G → GL n () : ρ (g) = ρ (g). Then ρ is also a representation of
G, and χ is the character of ρ .

Q4) i) O  S4 , which has 5 conjugacy classes, and hence, 5 irreducible


characters. One character corresponds to the trivial representation,
and one to the sign representation, say χ1 and χ 2 .
So d1 = 1 = d 2 .

26
Study Guide-II (1) (1) (2) (2) (2)
1 −1 i j ij
χ1 1 1 1 1 1
Note that D4 and Q8 are
not isomorphic groups. χ2 1 1 1 −1 −1
Yet they have the same χ3 1 1 −1 1 −1
character table.
χ4 1 1 −1 −1 1
χ5 2 −2 0 0 0

The character table of D 4 is given in Example 1, and is the same as that


of Q8 , as you can see.

Q12) Let’s say χ5 is the missing character corresponding to an irreducible


representation ρ5 . Then G = 24 = 12 + 12 + 32 + 32 + d52 , so that d 5 = 2.
So χ5 (1) = 2.
Then, using orthogonality of the columns in the table, we get
χ5 (a) = 2, χ5 (b) = 0, χ5 (c) = 0, χ5 (d) = −1.

-----------------------------------------------------------------------------------------------

Solutions to Questions 1, 2, 4, 6 of Section 8 on Page341, 342 of the


textbook.

Q1) The operation is pointwise multiplication.


Firstly, χ (g) ∈ * ∀g ∈ G and abelian characters χ . Next, χχ ′ is an
abelian character whenever χ and χ ' are.
Finally, the character of the trivial representation is the identity, and the
inverse of χ is defined by χ −1 (g) = [χ (g)] .
−1

Q2) From Example 2, the character group of K 4 is of order 4. So it is either


C4 or K 4 .

However, if we look at the representations, they are obtained by setting


( ρ (x) , ρ (y) ) to be (1, 1) , ( −1, 1) , (1, − 1) , ( −1, − 1) . So, the
corresponding irreducible characters all satisfy χi2 = ( ± 1) = 1.
2

Accordingly, the character group must be K 4 .

With reference to Q4 (ii), P.338, the character group is of order 4.


Arguing on the same lines as above, we see that the character group for
Q8 is also K 4 .

Q4) By Theorem 8.4, all its irreducible representations are one-dimensional.


Also, since G is abelian, each conjugacy class has cardinality 1. So the
number of distinct conjugacy classes is G . Hence the result.

Q6) a) G has n conjugacy classes, and hence n one-dimensional irreducible


representations.
2 πki
Each ρk (x) = e n , so that χk = 1.

28
Hence each ρk is irreducible. Characters

b) The character group of G is Cn .


1 ⎡ n −2 πim α 2πik α ⎤
c) For k ≡⁄ m (mod n ), χ m , χ k =
n

⎢ e n e n ⎥ = 0 , since the
⎢⎣α =0 ⎥⎦
sum is 1 + ξ + ξ2 + K + ξn −1, where ξ is a primitive nth root of unity.

29
Study Guide-II
UNIT 9 FIELDS

Structure Page
9.1 Introduction 30
Objectives
9.2 Field Extensions 30
9.3 Finite Fields 34
9.4 Summary 34
9.5 Solutions/Answers 35

Based on Chapter 13, Sections 1, 2, 3, 5, 6 of the book ‘Algebra’ by


M. Artin.

9.1 INTRODUCTION

In this unit, we shall help you study some concepts about fields and field
extensions. We will also be discussing different kinds of elements that extend
fields, and when two extensions are isomorphic. You would be familiar with
some of this, but taking a re-look will help set the tone for further study of
Galois theory.

Objectives

After studying this unit, along with Sections 1, 2, 3, 5, 6 of Chapter 13 of the


textbook, you should be able to:
• define, and give examples of, field extensions in any characteristic;
• obtain a splitting field of a polynomial;
• prove, and apply, the statement that a field is finite iff it is a splitting
n
field of x p − x over Fp for some prime p and some n ∈ N .

9.2 FIELD EXTENSIONS

In your undergraduate studies you have come across fields like Q, R and C .
Also Q ⊆ R ⊆ C . Thus, as you can see from Section 1, Chapter 13, C is an
extension field of R as well as of Q . For more on this

read Section 1 of Chapter 13 in the text, up to the line before (1.3).


Then read the following.
NOTE Recall the concept of the characteristic of an integral domain from the
IGNOU course “Abstract Algebra”, MTE-06. You would remember that the
characteristic of an integral domain is either ‘zero’ or a ‘prime’. Since every
field is an integral domain, the characteristic of a field is also either ‘zero’ or a
‘char R’ denotes ‘prime’. Further, any subfield of a field F has the same characteristic as F .
‘characteristic of R’.
Example 1: What are the characteristics of Q, C, F p ?

Solution: For Q, n.1 ≠ 0 ∀ n ∈ N . Therefore, char Q = 0 .


30
Since Q ⊆ C , char C = 0 . Fields

Finally, char F p = p (why?)


***

From Example 1, we conclude that all number fields are of characteristic 0,


whereas every finite field has characteristic p for some prime p .

Try the following exercises now.

E1) Let R denote an integral domain and F its field of quotients. What is EXERCISES
the characteristic of R [x 1 , x 2 , K , x n ] and of F[x 1 , x 2 , K , x n ] ?

Do Exercises 1, 3, 4 of Section 1, at the end of Chapter 13 in the book.

Following Q3 in the exercises above, consider the following definition.

Definition: Let K / F be a field extension. If K is a finite-dimensional vector


space over F , then we say K is a finite extension of F. Further, the dimension
of K as a vector space over F is called the degree of K/F, and is denoted
by [K : F] .

Now read Section 2, Chapter 13. While reading it, note the following
points.

i) Every element of a field K is algebraic over K. NOTE

ii) The irreducible polynomial for α over F is unique, and is also called the
minimal polynomial of α over F .

iii) Read the following before reading Proposition 2.5:

Regarding (2.3), there is yet another description of the field F(α) . Let
F denote the collection of all subfields L of K that contain F and α .
F ≠ φ as K itself is a member of F . Also F(α) ∈ F .
Next, the intersection T of all such subfields is a subfield of K that
contains F and α , and therefore contains F(α) . Consequently,
T = F(α) .
So, F(α) = ∩{L | L is a field extension of F, α ∈ L}.

iv) F(α) = F [α] iff α is algebraic over the field F. To see this, note that
F[α] is an integral domain and it is enough to show that every element
in F[α] has an inverse. Let g( α) ∈ F[α] , g(α) ≠ 0 . Then, the
irreducible polynomial of α over F , does not divide g . Also, since f
is irreducible over F , (g, f ) = 1 . So there are polynomials
p(x), q(x) ∈ F[x] such that g( x )p( x ) + f ( x )q ( x ) = 1 . Substituting α for
x and noting that f (α) = 0 , we get g(α )p(α ) = 1. So,
[g(α)]−1 = p(α) ∈ F[α] .

31
Study Guide-II v) Regarding (2.8), note that [F(α) : F] = [F(β) : F] does not imply
~ F(β) . e.g., consider Q [i] and Q [ 2 ] . Both have degree 2
F(α) −
over Q but are not isomporphic.

Definition: An extension K of a field F is called a simple extension of F if


K = F(α) for some α ∈ K . Here α is called a primitive element of K over F.

Why don’t you try the following exercises now!

EXERCISES E2) Prove Proposition 2.6 (b).

E3) Show that if a quadratic or cubic polynomial over a field F is not


irreducible, it has a root in F. Give an example to show that this need not
be true for polynomials of higher degree, i.e., ∃ f ∈ F[ x ] with deg f > 3
and f reducible over F such that f has no root in F .

Do Exercises 1-5, Section 2, at the end of Chapter 13 in the book.

NOTE Now read Section 3 of Chapter 13 of the textbook. While reading it note
the following points.

i) The term ‘invariant’ in the first paragraph of the section indicates that
given any two F -isomorphic field extensions, their degree is the same.

ii) In Proposition 3.3, where have we used the fact that char F ≠ 2 ? Not to
show that K = F [α] , where the irreducible polynomial of α over F is of
degree 2, but to show that K = F [δ] , where δ 2 ∈ F . Therefore, even if
char F = 2 , and [K : F] = 2 , then K = F[α] for some α satisfying an
irreducible polynomial x 2 + bx + c ∈ F[ x ] . However, in characteristic 2,
we may not be able to ‘complete the square’ and assume that ∃ δ ∈ F s.t.
f ( x ) = x 2 − δ . For example, F22 cannot be generated by δ over F2 for
any δ ∈ F2 , because δ = δ for any δ ∈ F2 .

iii) Corollary (to Corollary 3.6 of the textbook): Every extension of finite
degree of a field F is an algebraic extension.

Note that the converse of this corollary need not be true, as the following
example shows.

Example 2: Let q n denote the n th prime. Show that Q ( 2 , 3, K, q n , K)


is algebraic over Q , but not a finite extension of Q .

Solution: q n is algebraic over Q since it satisfies the irreducible polynomial


x 2 − q n . Let K = Q ( 2, )
3 , K , q n , K . Write
K 0 = Q, K 1 = Q ( 2), K 2 = K1 ( 3 ) , K, K n = K n −1 ( q ), K.
n

Then, for each n, [K n : K n −1 ] = 2 , so that K n / K n −1 is algebraic. By (3.11),


32 K n is algebraic over Q .
Now, for any α ∈ K , α ∈ K n for some n , and hence is algebraic over Q . Fields

Thus, K / Q is algebraic.
Since the number of primes is infinite, [K : Q] cannot be finite.
***

Try the following exercises now.

E4) Let K = {α ∈ C | α is algebraic over Q} . Show that K / Q is algebraic EXERCISES


but not finite.

Do Exercises 1, 2, 5, 7-10, 14 of Section 3, at the end of Chapter 13 of


the textbook.

Now read Section 5 of Chapter 13 of the book up to the proof of


Proposition 5.3. While doing so, please note the following.

i) In the proof of Lemma 5.2, consider the map NOTE


φ : F → F[x] : φ(a ) = a + < f > .
<f >
Note that φ is a field homomorphism which is injective
(a + < f >=< f >⇔ a ∈< f >⇔ a = 0 , since deg f > 0.)
This is why F can be treated as a subfield of F[ x ] .
<f >

ii) f ∈ F[ x ] has a root in K = F [x ] , because f ( x ) = f ( x ) = 0 in K.


<f >

iii) You know that every polynomial of degree n over Q, R or C can be


written as a product a 0 (x − α1 ) ( x − α 2 )K ( x − α n ) , with
a 0 , α1 , K , α n ∈ C . In fact, this is true more generally, as Proposition
(5.3) tells us.

Proposition (5.3) leads us to the following definition.

Definition: A field extension K / F is called a splitting field of f ( x ) ∈ F [ x ] if it


is the smallest extension in which f can be written as a product of linear factors,
i.e., K is the smallest extension of F which contains all the roots of F.

If α1 , α 2 , K , α m are the roots of f ( x ) ∈ F[ x ] , then F(α1 , α 2 , K , α m ) is a NOTE


splitting field of f.

Try the following exercises now.

E5) Let F be a filed, f be a polynomial of degree n over F[ x ] and K be the EXERCISES


splitting field of f . Show that [K : F] ≤ n !.

We will now focus our attention on fields with finitely many elements only.

33
Study Guide-II
9.3 FINITE FIELDS

In this section we will discuss fields that are finite-dimensional Fp -vector


spaces, for some prime p. Thus, these fields consist of only finitely many
elements, and are called finite fields, or Galois fields. Such fields were first
studied by Evariste Galois (Pronounced Ga-low-aa) in 1830. As you will see in
Unit 10, and in the later courses, Galois fields are essential for many
applications.

Now read Section 6, Chapter 13 of the textbook.

NOTE While doing so, please read the following notes too.
i) (6.18) is a more general statement than (6.4) (c). Thus, (6.4) (c) is
proved as a particular case of (6.18), where H = F* .
ii) To prove H is cyclic in (6.18) you use the Structure Theorem (see Unit
4, Section 4.4 of Study Guide-I).
iii) The essence of this section is the following result.

Theorem: A field F is finite, with p n elements, iff F is a splitting field


n
of x p − x over Fp .

Sketch of Proof: Firstly, assume F is finite. Then, from (6.4) (c) and (d)
n
of Section 6 of the textbook, you see that F is a splitting field of x p − x
over Fp .
n
Conversely, if F is a splitting field of f ( x ) = x p − x over Fp , it is of the
form Fp (α1 , K , α r ) , where α i are the distinct roots of f ( x ) . Use
Proposition (5.7) of Section 5, Chapter 13, now to note that r = p n .
Now, Proposition (6.19) (b) tells us that {α1 , K , α p n } is a field. Thus,
F = {α1 , K , α pn } . So F is a field with p n elements.

iv) Proposition (6.4) (b) tells us that any two fields of order p n are
isomorphic.

Try the following exercises now.

EXERCISES Do Exercises 2-5, 7, 9, 10, 15 of Section 6 at the end of Chapter 13 of the


textbook.

With this we come to the end of this unit. In the next unit you will be using the
facts studied here. For now, let us summarise the points taken up in this unit.

9.4 SUMMARY

In this unit we have covered the following points:

1. i) A field K is an extension of a field F, if F is a subfield of K .


34
ii) K is called an algebraic extension of F if each element of K is Fields
algebraic over F, i.e., it satisfies some non-zero polynomial in
F[x] .
iii) An element of K is transcendental over F if it is not algebraic
over F.

2. α is algebraic over F iff F[α] ~_ F(α) ~_ F[x ] , where f ( x ) is


< f (x) >
the minimal polynomial of α over F.
α is transcendental over F iff F [α] ~_ F [ x ] .

3. If K / F is a field extension such that K is a finite-dimensional vector


space over F, then K / F is called a finite extension. [K : F] denotes the
dimension of K over F, and is called the degree of K over F. If α is
algebraic over F, then [F(α) : F] = degree of the irreducible polynomial
of α over F.

4. If L / K and K / F are finite extensions, then L / F is also finite and


[L : F] = [L : K ] [K : F] .

5. Any polynomial in F [ x ] has a splitting field over F , that is the smallest


field extension of F that contains all the roots of f. If the roots of f are
α1 , K , α n , then a splitting field of f over F is F(α1 , K , α n ) .

6. If F is a finite field, then | F | = p n for some prime p and some integer


n ≥1.

7. If F is a field and H is a subgroup of F* such that | H | = n , then H is a


cyclic group and it consists of all the n th roots of unity in F.

n
8. F is a finite field iff it is a splitting field of x p − x ∈ p [x ] over p for
some prime p and some integer n ≥ 1 .

9.5 SOLUTIONS/ANSWERS

E1) Case I: char R = 0 ⇒ char F = 0 ⇒ characteristic of both rings given


is 0.
Case II: char R = p . Similarly, characteristic of both rings given is p.

E2) We prove this by induction on n . It has already been proved for n = 1


in (2.6) (a). Assume it is true for n = m , i.e.,
f [α1 , K , α m ] = F (α1 , K , α m ) = K , say.
Now F[α1 , K , α m , α m+1 ] = F [α1 , K, α m ] [α m +1 ] = K [α m +1 ] .
Since α m +1 is algebraic over F , it will be algebraic over K . So,
applying (2.6) (a), we find
K [α m +1 ] = K (α m+1 ) = F(α1 , α 2 , K , α m , α m+1 ) .
Hence the statement is true for all n ≥ 1 .

35
Study Guide-II E3) A polynomial f is reducible if f = gh , where deg g. deg h ≥ 1 .
If deg f = 2 , this can happen if deg g = 1 = deg h , i.e.,
f = (ar + b) (cr + d ) , i.e., if f has roots in F .
If deg f = 3, f can be reducible if f = gh with either deg g = 1 or
deg h = 1 . Accordingly, f has a root in F .
For the example, take f = ( x 2 + 1) ( x 2 + 2) ∈ Q [ x ] . f is reducible over
Q , but has no root in Q .

Solutions to Exercises 1, 3, 4 of Section 1, at end of Chapter 13 of the


textbook.

Q1) If a ∈ F is such that a = a −1 , then a 2 = 1 . The equation x 2 − 1 = 0 has at


most 2 roots, which are ±1 if the characteristic of F is not 2, and the
only root is 1 if the characteristic of F is 2.

Q3) Let R have dimension n over F and let 0 ≠ a ∈ R .


{ }
Then the set 1, a , K , a n of n + 1 elements is linearly dependent over
F. So, there exist α 0 , α1 , K , α n ∈ F , not all zero, such that
α 0 + α 1a + L + α n a n = 0 .
Suppose α 0 ≠ 0 . Then, we have
{( − α −1
0 α1 ) + (− α −1
0 α2 ) a + L + (− α −1
0 αn ) a } a = 1 , showing a is
n −1

invertible.
If α 0 = 0 , we can find the first non-zero α i , say α k and use a similar
argument to obtain an inverse of a k , and hence an inverse of a.
Thus, each non-zero element of R is invertible, and hence R is a field.

Q4) The field has characteristic 2. Why?


Note that 8 × 1 = 0 , so that (2 × 1) (2 × 1) (2 × 1) = 0 in F, so that 2 × 1 = 0 .

Solutions to Exercises 1-5 of Section 2, at end of Chapter 13 of the textbook.

Q1) Since α is a real cube root of 2, α 3 = 2 .


Write y = 1 + α 2 . Then y 2 = 1 + 2α 2 + α 4 = 1 + 2α + 2α 2 , since
α3 = 2 .
⇒ ( y − 1) 2 = 2α , using α 2 = y − 1 .
⇒ ( y − 1) 4 = 4α 2 = 4( y − 1) ⇒ y 3 − 3y 2 + 3y − 5 = 0 , since y − 1 ≠ 0 .
You can see y satisfies the equation x 3 − 3x 2 + 3x − 5 = 0 , which is
p
irreducible over Q . Why? [If ∈ Q is a root of
q
a n x n + a n −1x n −1 + L + a 1x + a 0 ∈ Z [ x ] , with (p, q ) = 1 , then p | a 0 and
q | a n . So, in this case you only need to check that ±5 is not a root of
the given equation.]

Q2) Show that the given set is linearly independent over F using the fact that
the polynomial over F of least degree satisfied by α is of degree n.
Here we assume f ( x ) = a n x n + a n −1 x n −1 + L + a 0 , with f (α) = 0 .
36
To show that the set {1, α, α 2 , K, α n −1} spans F[α] , consider any Fields

element g(α) of F[α] . By the division algorithm,


g( x ) = f ( x ) q (x ) + r ( x ) , for some q, r ∈ F[x ] with r( x ) = 0 or deg r < n .
{
If deg g < n , then g(α) is in the span of 1, α, K , α n −1 . }
If deg g ≥ n , then g(α) = r (α) , since f (α) = 0 . Thus, again g(α) is in
the span of the given set.
{ }
Hence 1, α, K , α n −1 is a basis for F [α] over F .

Q3) Let α = 3 + 5 .
(a) A polynomial over Q that has α as a root, also has
3 − 5 , − 3 + 5 and − 3 − 5 as its roots. Thus,
(x − ( 3+ 5 )) (x + ( 3+ 5 )) (x − ( 3− 5 )) (x + ( 3− 5 ))
= x − 16 x + 4 is the irreducible polynomial of α over Q .
4 2

( )
(b) As F = Q 5 , 5 ∈ F . Therefore, if z = α − 5 = 3 , then
z = 3 , i.e., α 2 − 2 5α + 5 = 3 . So x 2 − 2 5x + 2 is the irreducible
2

polynomial of α over F.

(c) Check that this will be the same as the polynomial over Q , since any
polynomial of lesser degree will have 5 or 15 as coefficients.

(d) [x − ( 3 + 5 ) ] [ x + ( 3 + 5 ) ] is the required polynomial.

Q4) Since α is a root of x 3 − 3x + 4, α 3 = 3α − 4 …(i)


Therefore, α 4 = 3α 2 − 4α …(ii)
( )(
Now α 2 + α + 1 cα 2 + bα + a = 1 )
⇒ α 2 (a + b + 4c) + α(a + 4b − c) + (a − 4b − 4c − 1) = 0 , using (i)
and (ii).
Since α cannot satisfy an equation over Q of degree less than 3,
a + b + 4c = 0, a + 4b − c = 0, a − 4b − 4c − 1 = 0 .
Solving these equations for a, b, c, we obtain the inverse as
1
49
(
17 − 5α − 3α 2 . )
Q5) Since α is a root of f ( x ) = x n + a n −1 x n −1 + L + a 0 , and f ( x ) is
irreducible, a 0 ≠ 0 . So, α n + a n −1α n −1 + L + a 0 = 0 .
( )
⇒ α − a 0−1α n −1 − a 0−1a n −1α n − 2 − L − a 0−1a 1 = 1
⇒ α −1 = − a 0−1α n −1 − a 0−1a n −1α n −2 − L − a 0−1a 1 .

E4) By (3.10) of the textbook, K is a field. By the way k is defined, K / Q


is algebraic. Now, for any n ∈ N , x n − 2 is irreducible over Q . So
[Q ( 2 ): Q] = n . Therefore, [K : Q] ≥ n . Thus, K / Q is not finite.
n

37
Study Guide-II Solutions to Exercises 1, 2, 5, 7, 8, 9, 10, 14 of Section 3, at the end of
Chapter 13 of the textbook.

Q1) [F[α ] : F] = 5 = [F [α] : F[α 2 ]] [F [α 2 ] : F]


Since α 2 ∉ F, [F[α 2 ] : F] > 1 . Hence, F[α 2 ] = F [α]

Q2) (
ξ satisfies x 7 − 1 = (x − 1) x 6 + x 5 + x 4 + x 3 + x 2 + 1 over Q , ξ ≠ 1 )
and x + x + L + 1 is irreducible over Q .
6 5

Thus [Q(ξ) : Q] = 6 . Similarly, [Q(η) : Q] = 4 .


If η ∈ Q(ξ) , then Q(η) ⊆ Q(ξ) , so that
[Q(ξ) : Q] = [Q(ξ) : Q(η)] [Q(η) : Q]. But 4 does not divide 6.
Hence η ∉ Q(ξ) .

Q5) [K : F] = 1. Let {β} be a basis of K over F, i.e., K = Fβ . Since


1 ∈ F ⊆ K , ∃α ∈ F such that 1 = αβ . Thus, β = α −1 ∈ F . Therefore,
K ⊆ F . Hence K = F .

Q7) ( )
i) Now if i ∈ Q − 2 , then Q(i) = Q − 2 , so that ( )
i 2 ∈ Q(i) ⇒ i 2 = α + β i , where α, β ∈ Q .
This is a contradiction. Hence i ∉ Q − 2 . ( )
ii) Next, suppose i ∈ Q (− 2 ) [ 1/ 4
]. Then [Q[(− 2) ]: Q[i]] = 2 , since
1/ 4

[Q (−2)1 / 4 : Q] = 4 . Therefore, (− 2)1 / 4 satisfies x 2 + αx + β , where


α, β ∈ Q[i] .
∴ α = a 0 + a 1i, β = b 0 + b1i, a 0 , a 1 , b 0 , b1 ∈ Q .
Now, putting (−2)1 / 4 in the polynomial, we get
i 2 + α(−2)1/ 4 + β = 0 ⇒ i 2 + β = −α(−2)1 / 4
⇒ −2 + b 02 − b12 + 2b 0 b1i − 2b1 2 + 2 2b 0i = (a 20 − a 12 )i 2 − 2 2a 0 a 1 ,
on squaring.
Comparing coefficients of i first, and then comparing coefficients of
2 we get b 02 − b12 = 2, a 0 a 1 = b1 , b 0 b1 = 0, 2b 0 = a 20 − a 12 .
b 0 b1 = 0 ⇒ b 0 = 0 or b1 = 0 . But then b12 = −2 , a contradiction, or
b 02 = 2 , again a contradiction.
[
Therefore, i ∉ Q (− 2)1 / 4 . ]
iii) As in Q2 above, using the facts that [Q(i) : Q] = 2 and x 3 + x + 1 is
irreducible over Q , you can show that i ∉ Q(α) .

Q8) K = F(α, β) , where deg α = m, deg β = n, (m, n ) = 1 .


Then F ⊆ F(α) ⊆ K , and F ⊆ F(β) ⊆ K .
So [K : F] = [K : F(α)] [F(α) : F]
= [K : F(β)] [F(β) : F]
So m [K : F] and n [K : F] .
Thus, mn | [K : F] , since (m, n ) = 1 .

38
Also deg β over F(α) is at most n, since it is at most n over F. So Fields

[K : F (α)] = [F (α) (β) : F (α)] ≤ n . Therefore, [K : F] ≤ mn .


Hence [K : F] = mn .

Q9) If β ∈ Q(α) , then [K : Q] = 3 .


If β ∉ Q(α) , then deg β over Q(α) is again 3, so that [K : Q] = 9 .

Q10) If α + β and αβ are algebraic over Q , then (α − β) 2 = (α + β) 2 − 4αβ


An algebraic number
is also algebraic over Q . is an element of C
Therefore, α − β is algebraic over Q . which is algebraic over
Q.
Therefore, α and β are algebraic over Q .
[Note: You can also use Theorem 3.11 of the textbook to prove this.]

{ }
Q14) You can prove that α i β j | 0 ≤ i ≤ d − 1, 0 ≤ j ≤ e − 1 spans F[α, β] over
F.
If degree of β over F[α] is e, this set will be linearly independent.
Otherwise, it need not be linearly independent, and hence it won’t form a
basis over F .
e.g., ω(= e 2πi / 3 ) and ω2 are algebraic over Q , both of degree 3.
But Q[ω, ω 2 ] = Q[ω] has degree 3 over Q .

E5) The proof is by induction on n . If n = 1, f ( x ) is linear and K = F . So,


[K : F] = 1 ≤ 1! .
Suppose n > 1 . Let α be a root of f ( x ) , and L = F[α] . Then
[L : F] ≤ n . Write f ( x ) = ( x − α) g ( x ), g ( x ) ∈ L [ x ] . Then, g( x ) has
degree n − 1 . So, by induction, the degree of the splitting field of g( x ) ,
say, E over L is at most (n − 1)! . Now,
E = L (α1 , K, α n −1 ) = F(α1 , α 2 , K, α n −1 ) = K . Further,
[K : F] = [K : L] [L : F] ≤ (n − 1)!n = n! . Hence result is true for all n ≥ 1 .

Solutions to Exercises 2-5, 7, 9, 10, 15 of Section 6 at the end of


Chapter 13.

Q2) If {1, α} is a basis of F4 over F2 , then F4 is {0, 1, α, 1 + α} , with


α 2 + α + 1 = 0 , since F4 consists of the roots of x 4 − x ∈ F2 [ x ] . Write
the Cayley tables for both sets, w.r.t. the two operations. Compare their
properties, in terms of whether the identities exist, invertibility,
commutativity, associativity, etc.

Q3) 3 ∈ F13* . Therefore, 312 = 1 .


∴ 313 = 3 .
Thus, 3 ∈ F13 is a 13th root of 3.

Q4) As in (6.12) of the textbook,


F23 = {0, 1, β, 1 + β, β 2 , 1 + β 2 , β + β 2 , 1 + β + β 2 } , where β is a root of
39
Study Guide-II
f (x) = x 3 + x + 1.
Let g( x ) = x 3 + x 2 + 1 . Then f ( x + 1) = g( x ) . You should see why the
irreducible polynomials over F2 for each are
x , x + 1, f (x ), g (x ), f ( x ), g( x ), g( x ), f ( x ) , respectively.

3
Q5) Any irreducible polynomial of deg 3 over F3 must divide x 3 − x , by
Theorem 6.4 (e). So all its roots are in F27 .
Since 3 is prime, every α ∈ F27 \ F3 satisfies an irreducible polynomial.
There are 24 such elements. Since each irreducible polynomial has 3
24
distinct roots, there are = 8 distinct cubic polynomials over F3 .
3

Q7) i) x 9 − x = x (x − 1) (x + 1) (x 2 + 1) ( x 4 + 1)
= x ( x − 1) ( x + 1) ( x 2 + 1) ( x 2 − x − 1) (x 2 + x − 1) .
You can check that each of these factors is irreducible over F3 .

ii) We use the solution to Q5 to obtain the factorisation of x 27 − x


over F3 . There are 8 irreducible polynomials of degree 3. Since
these cover the 24 elements not in F3 , these are the only non-
linear irreducible polynomials. Thus the factorisation is
x (x − 1) ( x − 2) ( x 3 + x + 1) (x 3 − x + 1) ( x 3 + x − 1) (x 3 − x − 1)
( x 3 + x 2 + 1) ( x 3 − x 2 + 1) ( x 3 + x 2 − 1) ( x 3 − x 2 − 1) .

Q9) Firstly, f ( x ) = x q − x is one such polynomial.


If g( x ) is any other such polynomial, then x q − x = ∏ ( x − α) must
α∈Fq

divide it. So the required polynomials are all multiples of f ( x ) .

−1
= ∏ (x − α i ) .
n
Q10) Let | K | = p n . Then x p
α i ∈K

Putting x = 0 in this gives (−1) p


n
−1
(∏ α i ) = −1 .
For p ≠ 2 , p n − 1 is even, and hence the result.
If p = 2, − 1 = 1 , and hence the result.

Q15) K = F(α), L = F(β) , where α 3 + α + 1 = 0, β 3 + β 2 + 1 = 0 .


{1, α, α 2 } and {1, β, β 2 } are F-bases of K and L, respectively.
Define φ : K → L : φ(a + bα + cα 2 ) = a + bβ + cβ 2 ∀ a , b, c ∈ F , and
extend it to form a vector space isomorphism.
However, φ is not a field isomorphism. (See (2.9) of Chapter 13 of the
textbook here).
On the other hand, as noted above in the solution to Q4, if α is a root of
f , α + 1 will be a root of g . Thus, K = L . Thus, the identity map is an
explicit isomorphism from K to L .

40
Galois Theory
UNIT 10 GALOIS THEORY

Structure Page
10.1 Introduction 41
Objectives 42
10.2 Some Types of Extensions
Separable Extensions
Normal Extensions
Galois Extensions
10.3 The Fundamental Theorem of Galois Theory 47
10.4 Summary 47
10.5 Solutions/Answers 48

Based on Section 5, Chapter 13 and Sections 1, 4 and 5 of Chapter 14 of


the textbook.

10.1 INTRODUCTION

In the previous unit you studied about finite extensions. You found that any
such extension is algebraic. Here we will consider finite extensions that are
splitting fields of polynomials. The main focus of this unit is the Fundamental
Theorem of Galois Theory. This theorem is a very important tool, as it allows
us to study various aspects of finite extensions by considering equivalent
problems given in terms of groups, which are sometimes easier to solve.

Galois Theory has its origin in a classical problem in the theory of equations,
namely, “can the roots of a polynomial equation of degree ≥ 5 be obtained by
radicals and the basic arithmetic operations only?” This is named after the
French mathematician Galois, who studied symmetrics of roots of polynomials,
and published three papers that laid the foundations for Galois theory.
Fig.1: Evariste Galois
The approach to Galois theory, through the use of automorphisms of field (1811-1832)
extensions, is largely developed by Dedekind, Kronecker and Emil Artin. We
will be focusing on this approach in this unit.

In the next unit you would be studying some applications of this theory.

Objectives

After studying Section 5, Chapter 13 and Sections 1, 4, 5 of Chapter 14, with


this unit as a guide, you should be able to:
• explain what a separable field extension is, and give examples of this;
• define a normal field extension, and give examples of this;
• define a Galois extension;
• prove, and apply, some properties of separable/normal/Galois
extensions;
• state, explain and prove the Fundamental (Main) Theorem of Galois
Theory.

41
Study Guide-II
10.2 SOME TYPES OF EXTENSIONS

In this section we focus on three kinds of extensions, namely, normal,


separable and Galois extensions.

10.2.1 Normal Extensions

In the previous unit you studied that every polynomial has a splitting field. In
fact, upto K-isomorphism, this field is unique for a given polynomial over a
field K. This is a corollary of the following theorem, which we shall only state.
∃η
K ~ K
1 2 Theorem 1: Let F1 and F2 be fields, σ : F1 → F2 a field isomorphism and
f ( x ) ∈ F1 [x ] . Let K 1 and K 2 be splitting fields of f and σ (f ) over F1 and
F2 , respectively. Then there exists an isomorphism η : K 1 → K 2 such that
U σ U
F
~ F η | F1 = σ . Further, the number of such isomorphisms is at most [K1 : F1 ] .
1 2

Fig.2: A commutative
diagram representing the
Let us now consider a related definition.
situation in Theorem 1.
Definition: A finite field extension K / F is called a normal extension if it is a
splitting field of some polynomial over F.

For example, C / R is normal since C is the splitting field of x 2 + 1∈ R[ x ] .


n
Similarly, any finite field is normal, being the splitting field of x p − x ∈ Fp [ x ]
for some prime p and n ∈ N .

Now, there is something very interesting about a normal extension K / F . Not


only is it a splitting field of a given polynomial over F, but any other
irreducible polynomial over F with one root in K also splits in K. Let us see
why.

Theorem 2: Let K / F be a finite extension. Then the following are


equivalent.
i) K / F is a normal extension.
ii) For any F-homomorphism σ : K → L , where L / K is an extension,
σ(K ) = K .
iii) If f ( x ) ∈ F [ x ] is irreducible and has one root in K, then all its roots lie
in K.

Proof: We will prove they are equivalent by showing (i) ⇒ (ii ) ⇒ (iii ) ⇒ (i) .

i ⇒ ii : Let K = F (α1 , K , α n ) be the splitting field of f ( x ) over F, with


roots α1 , α 2 , K , α n . Since σ is injective and σ(α i ) is also a
root of f ( x ) , σ merely permutes the roots. Thus, σ(K ) = K .

ii ⇒ iii : Let f ( x ) be irreducible over F [ x ] with one root α ∈ K . Let β be


another root of f and let L be a splitting field of f over K. Then by
Proposition 2.9, Chapter 13 of the textbook, ∃ an F-isomorphism
φ : F (α) → F(β) : φ(α) = β .

42
Then, by Theorem 1, φ can be extended to an F-isomorphism Galois Theory
~ L . So, σ | : K → L is an F-homomorphism, and
σ : L ⎯⎯→ K
hence, by our hypothesis, σ(K ) = K . Thus, σ(α) = φ(α) = β ∈ K .

iii ⇒ i : Since K / F is finite, let K = F (α1 , α 2 , K , α r ) . Let f i ∈ F [x ] be


an irreducible polynomial with one root α i ∀ i = 1, K , r , and
r
f = ∏ f i . Then, by our assumption, all the roots of f are in K.
i =1
Further, the way K is defined, it is the smallest such field, and hence
the splitting field of f . Hence K / F is normal.

Note: A version of Theorem 2 also holds true for any algebraic extension NOTE
K / F . The difference is only that the definition of a normal extension changes
to ‘ K / F is normal if it is a splitting field of a set of polynomials.’ For more
on this you can refer to Section 3, Chapter 5 of ‘Algebra’ by Hungerford, for
example.

However, we shall restrict our discussion to finite extensions only.

Try the following exercises now.

E1) Show that any field extension of degree 2 is normal. EXERCISES

E2) If ξ is a primitive nth root of unity, then show that Q (ξ) / Q is a normal
extension.

E3) a) Prove that if K ⊆ E ⊆ L are fields such that L / K is a normal


extension, then L / E is also normal.
b) If K / F and L / K are normal extensions, will L / F be
normal? Give reasons for your answer.

Let us now look at separable extensions.

10.2.2 Separable Extensions

In the previous unit you have studied that if F is a field and f ( x ) ∈ F[ x ] is


irreducible, then there is an extension K of F over which f ( x ) splits
completely as a product of linear factors. Now, if these factors are distinct, we
have the following definition.

Definitions: i) If an irreducible polynomial f ( x ) ∈ F[ x ] , where F is a field, has


no multiple roots, then f ( x ) is called a separable polynomial.

ii) An element α of an extension K / F is called separable if its irreducible


polynomial over F is separable. (Note that this means α is algebraic over F.)

iii) An extension K of a field F is called a separable extension if α is


separable ∀ α ∈ K \ F .

43
Study Guide-II
For example, i is separable over Q (or R ). Note that x 2 + 1 is separable over
R , but not over F2 , since it has 1 as a multiple root in F2 .

Also, C \ R is separable, because for any z ∈ C \ R , its irreducible polynomial


over R is ( x − z) ( x − z ) .

Sometimes it is not easy to find all the roots of a polynomial, and hence to
decide if it is separable or not. One tool to help us is its derivative, which you
can now study about.

Read the matter from (5.5) in Section 5 of Chapter 13 till the end of the
section.

Try the following exercises now.

EXERCISES E4) Prove that if char F = 0 , an extension K / F is algebraic iff it is


separable.

E5) Show that for char F = p ≠ 0, f ∈ F [ x ] is separable iff it is not of the


form g( x p ) , where g ∈ F [ x ] .
[Hint: Show that f ( x ) = g ( x p ) iff f ′ = 0.]

We now prove a result of theoretical and practical importance.

Theorem 3: A finite separable extension K / F is simple.

Proof: Any finite extension is of the form F(α1 , K , α n ) . Let us first prove
the result for K = F (α, β) , then the general result will follow by induction.

So, assume K = F(α, β) . If F is finite, so is K. Then K * will be cyclic. If


K * =< r > , then K = F(r) .

Now suppose F is infinite. Let the irreducible polynomials of α and β over F


be f ( x ) and g( x ) , respectively. Let the roots of f and g be
α = α1 , α 2 , K , α m and β = β1 , β 2 , K , β n , respectively.

Now, study the proof of Theorem 4.1, and Example 4.3, Section 4, Chapter
14 of the textbook.

Try the following exercises now.

EXERCISES E6) Prove that if K / F is separable, so is E / F ∀ fields E such that


F⊆ E⊆K.

E7) a) Let K be a field. Then show that the set of all automorphisms of K
forms a group G with respect to composition of maps. We
denote G by Aut K.

44
b) Let K / F be a field extension. Then show that the set of all Galois Theory
F-automorphisms of K forms a group under the composition of
maps, and this is a subgroup of Aut K.

E8) Show that Fpn is a separable extension of Fp , where n ≥ 1 .

E9) Show that a finite extension L / K is separable iff there are [L : K ]


distinct K-homomorphisms of L into N , where N is any normal
extension of K containing L .

E10) Prove that if L / K and K / F are separable extensions, then L / F is


separable.

E11) Show that if α is separable over F, then F(α) / F is separable.

Now we shall discuss, field extensions that are the focus of Galois theory.

10.2.3 Galois Extensions

We introduce you to a particular kind of finite field extension now, named after
Galois.

Study Section 1, Chapter 14, of the textbook from the beginning upto the
point before (1.10).

NOTE
While studying it, note the following:
i) If α and β are the roots of an irreducible quadratic polynomial over F,
then F(α) = F(β) = K, say.
σ : K → K : σ(a + bα) = a + bβ is an F-automorphism.
Suppose φ is any other F-automorphism of K, then
φ(a ) = σ(a ) ∀ a ∈ F.
Also, φ(α) is a root of the irreducible polynomial of α . Thus,
φ(α) = β. Hence, φ = σ. So, G (K / F) = {I, σ} .
ii) What is done in this section for fields of characteristic ‘zero’ also holds
for fields of characteristic p. The proofs require suitable modifications.
iii) In the book, a Galois extension is only defined for fields of
characteristic zero. However, the same definition holds for all fields.
iv) If K = F, G (K / F) = {I} . However, the converse is not true (see
Example 1 below).
v) Let K / F be a field extension and α ∈ K be algebraic over F, with
irreducible polynomial of degree n. Then any σ ∈ G (F(α) / F) is
completely determined by its action on α , since {1, α, K , α n −1 } is a
basis of F(α) over F. Also, since σ(α) is a root of
f ( x ) ∀σ ∈ G (F(α) / F) , | G (F(α) / F) | = m ≤ n , where m is the number
of distinct roots of f ( x ) in F(α) .

45
Study Guide-II Example 1: Give an example where | G (F(α) / F) | < [F(α) : F] .

Solution: Take K = R, F = Q and f ( x ) = x 3 − 2 . We know f ( x ) is


irreducible over Q . Let us take α = 3 2 . Then Q ⊂ Q(α) ⊂ R and
G (Q(α) / Q) = {I} , since Q(α) contains only one real root of the polynomial,
namely, α . So | G (Q(α) / Q) | = 1 < 3 = [Q(α) : Q] .
***

Example 2: Determine the Galois group, G (C / R) .


Solution: C = R[i] , and the irreducible polynomial of i over R is x 2 + 1 .
Thus, G (C / R) has at most two R -automorphisms in it. Also, σ : C → C ,
defined by σ (a + ib ) = a − ib , is an R -automorphism of C . So,
G (C / R ) = {I, σ} .
***

Now, in the textbook you have seen that a finite field extension K / F is called
a Galois extension if | G (K / F) | = [K : F] . The following theorem gives you an
equivalent definition.

Theorem 4: A finite field extension K / F is a Galois extension iff K / F is


separable and normal.

Proof: Firstly, let K / F be finite, separable and normal. By (1.6) of the


textbook, you know that | G (K / F) | ≤ [K : F] .
Now, since K / F is finite and separable, by Theorem 3, K = F(α) , for some
α ∈ K \ F . Let f ( x ) be the irreducible polynomial of α , and deg f = n . Then
[K : F] = n. Also, f has n distinct roots α = α1 , α 2 , K, α n in K .
Correspondingly ∃ distinct σ i ∈ G (K / F) .
Thus, | G (K / F) | = n = [K : F] .

Conversely, assume that K / F is a Galois extension. We need to show that


K / F is separable and normal. For this, let f ( x ) be an irreducible polynomial
over F with one root α in K , and let G (K / F) = {σ1 , σ 2 , K , σ n } . Then
{σ i (α)} is the set of roots of f ( x ) . So all the roots of f ( x ) are in K .
Therefore, by Theorem 2, K / F is normal.

Now, take any α ∈ K \ F , with its irreducible polynomial f ( x ) . Any


F-automorphism of K is an F-homomorphism of F(α) into K . So, the
number of such homomorphisms is | G (K / F) | = [K : F] . So, the number of
distinct roots of f ( x ) = [K : F] ≥ [F(α) : F] = deg f . Hence, f has all its roots
distinct. Thus, K / F is separable.

Theorem 4 leads us to the following definition.

Definition: A field extension K of a field F is called a Galois extension if it is


a finite, normal, separable extension.

46
For example, by E1, you know that any finite extension of a field of Galois Theory
characteristic zero is separable. So, in this case, K/F is Galois iff K/F is
normal iff K is a splitting field of a polynomial in F [ x ] .

Let us restrict our study, as in the textbook, to fields of characteristic zero,


henceforth.

Now study Section 1, Chapter 14, from Theorem (1.11) till the end of the
section.

Try Q 1-3, 6, 7, 9, 10, 13 of Section 1 of the Exercises of Chapter 14 of the EXERCISES


textbook.

Let us now go back a bit and focus on the main theorem of Galois Theory.

10.3 THE FUNDAMENTAL THEOREM OF GALOIS


THEORY

In Theorem (1.15), Section 1, Chapter 14, of the textbook, you read that given
a Galois extension K / F , there is a 1-to-1 correspondence between the
intermediate extensions and the subgroups of G (K / F) . Let us see why this is
so.

Let us note down the stages required for understanding the proof of Theorem
(1.15).

1) Any two splitting fields of a polynomial over a field F are isomporphic.

2) Let F ⊆ L ⊆ K , then the fixed field of G (K / L) is L . Further, if


H ≤ G (K / F) , then K H is an intermediate extension of F such that
K / K H is Galois and H = G (K / K H ) .

3) If F ⊆ L ⊆ L′ ⊆ K , then G (K / L′) ≤ G (K / L) . Thus, the


correspondence in (1.15) is order reversing.

Now study Section 5, Chapter 14 of the textbook.

Try Q 2-3, 12 of Section 5 of the Exercises of Chapter 14 of the textbook. EXERCISES

With this we come to the end of our discussion on Galois Theory. Let us
summarise what you studied in this unit.

10.4 SUMMARY

This unit complements parts of Section 5, Chapter 13 and Sections 1, 4, 5 of


Chapter 14 of the textbook. Though the discussion has largely been restricted
to fields with characteristic zero, we have also tried to give a flavour of what
happens over finite fields.
47
Study Guide-II While studying the unit, you would have considered the following points.

1. The definition, and examples, of a normal field extension.


2. The definition, and examples, of a separable field extension.
3. The definition, and examples, of a Galois extension.
4. The proof, and applications, of the statement that a finite separable
extension is simple.
5. K / F is a finite normal extension iff whenever one root of an
irreducible polynomial over F is in K then all its roots are in K .
6. K / F is a Galois extension iff it is finite, normal and separable.
7. The proof, and examples, of the fundamental theorem of Galois theory,
which states that if K / F is a Galois extension, then there is a bijective
map between the set of subgroups of G (K / F) and the intermediate
field extensions of K / F . Under this map the normal intermediate
extensions correspond to the normal subgroups of G (K / F) .

10.5 SOLUTIONS/ANSWERS

E1) Let K = F(α) , where α satisfies an irreducible quadratic polynomial f


over F, f = x 2 + bx + c , say. Let the other root of f be β , then
α + β = −b ∈ F ⊆ K . Thus, β ∈ K . Also, any field containing F, α and
β would contain K . Thus, K is the smallest field containing α and
β , and hence is the splitting field of f over F . Thus, K / F is normal.

E2) 1, ξ, ξ 2 , K , ξ n −1 are the roots of x n − 1 over Q . So Q(ξ) / Q is


normal.

E3) a) Since L / K is normal, L is the splitting field of some f ∈ K[x ] .


Since E ⊇ K , f ∈ E[ x ] too. ∴ L / E is normal.

b) ( ) ( )
This is not true. Let F = Q , K = Q 2 , L = Q 4 2 . Then
L and K are normal because they are quadratic extensions.
K F
However, L is not normal because L contains only the real
F
root of x − 2 .
4

E4) Let K / F be algebraic, α ∈ K and f ( x ) be its irreducible polynomial


over F . Then by Proposition (5.8), Chapter 13 of the textbook, f has
no multiple roots. Hence it is separable.
Conversely, if K / F is a separable extension, and α ∈ K , then there is
an irreducible polynomial over F that α is a root of. Hence α is
algebraic over F .

E5) Let f ∈ F[ x ] be of the form g( x p ) , i.e.,


f ( x ) = a 0 + a 1 x p + a i x 2 p + L + a n x np .

48
Then p | f ′( x ) . Hence f ′( x ) ≡ 0 , so that f has multiple roots. Thus, f Galois Theory
is not separable. So, f is separable implies that f is not of the form
g( x p ) .
Conversely, suppose irreducible f is not of the form g( x p ) , then one
of the terms in f will be of the form ax q where p /| q and p /| a . Then
f ′ will contain aqx q −1 , so that f ′ ≡/ 0 . Then, as in Proposition (5.8),
Chapter 13, if f and f ′ have common roots, f | f ′ , which is not
possible. Hence, f is separable.

E6) Let α ∈ E . Then α ∈ K , and hence its irreducible polynomial over F


is separable.
~
E7) a) If φ : K → K and ψ : K → ~
K , then ϕ o ψ : K →
~
K . Further, the
composition of maps is associative. Also, id : K → K is identity
w.r.t. o . ϕ −1 is the inverse w.r.t. o .
b) As in (a), you can show that ϕ o ψ is an F-automorphism
whenever ϕ and ψ are F-automorphisms. o is associative, id
is the identity element and ϕ −1 is the inverse of ϕ .

n
E8) Let α ∈ Fp n . The irreducible polynomial of α , say f , divides x p − x
n
(by Theorem (6.4), Chapter 13 of the textbook). Since x p − x doesn’t
have multiple roots, neither can f . Hence Fpn / Fp is separable.

E9) Let L / K be separable. Then L = K (α) for some α ∈ L . Let f be the


irreducible polynomial of α over K and deg f = n . Let N / L be a
normal extension. Since α ∈ N , all the roots of f are in N . So, for
each root α1 (= α), α 2 , K , α n of f , we can define
σ i : L → N : σ i (α ) = α i .
Each σ i defines a distinct K-homomorphism of L into N . Thus, there
are [L : K ] = n distinct K-homomorphisms from L into N .
Conversely, suppose the condition holds and α ∈ L . Let the irreducible
polynomial of α be f over K and its splitting field over L be N . Let
[L : K ] = n and σ1 , σ 2 , K , σ n be the distinct K-homomorphisms of L
into N . Then σ1 (α), K , σ n (α) are roots of f . The maximum
number of roots can only be [L : K ] since | G (L / K ) | ≤ [L : K ] . Thus,
all the roots of f are distinct, and hence f is separable.

E10) By Theorem 3, K = F(α), L = K (β) = F(α, β) for some α ∈ K \ F and


β ∈ L \ K . Let f and g be the irreducible polynomials of α and β
over F and K , respectively. Then, f and g split in any normal
extension of f containing L . Let their roots be α1 , K , α n and
β1 , K , β n , respectively.
Define F-homomorphisms σi : K → N : σi (α) = α i . Extend each to an
F-homomorphism σ′i : L = F(α, β) → N : σ′i (α) = α i and σ ′i (β) = β .

49
Study Guide-II Also define K-homomorphisms (and hence F-homomorphisms)
τ j : L = K (β) → N : τ j (β) = β j .
Then define mn distinct F-homomorphism φ ij : L → N by
φ ij (α) = σ ′i (α ) and φ ij (β) = τ j (β) ∀ i = 1, K , n , j = 1, K , m .
Thus, By E9, L / F is separable.

E11) Use E9 to show this.

Solutions to select questions of Section 1, Exercises, Chapter 14

Q1) See the portion following the proof of Proposition (4.4), Chapter 14, of
the textbook.

Q2) Q ⊆ Q(i) ⊆ Q(i, 2 ).


A basis of Q(i) over Q is {1, i} , since i 2 = −1 .
A basis of Q(i) ( 2 ) over Q(i) is {1, 2 } , since ( 2 ) 2 = 2 .
Thus, a basis of Q(i, 2 ) over Q is {1, i, 2, i 2 } .

Q3) G ( Q( 2 , 3 ) / Q) is the Klein-4 group, which has three proper non-


trivial subgroups. Thus, by Theorem (1.15), Q( 2 , 3 ) / Q has three
intermediate fields. They are Q( 2 ), Q( 3 ), Q( 6 ) .

Q6) a) Check that the splitting field over Q is Q(i) . So, the degree
is 2.
1 1 1
b) The roots of x − 2 are
3
23 , ω.2 3 , ω 2
.2 3 , where ω is a cube
⎛ 1 ⎞

root of unity. Thus, the splitting field over Q is Q ω, 2 3 ⎟.
⎜ ⎟
⎝ ⎠
⎧⎪ 1 1 2 2 ⎫⎪
Its basis over Q is ⎨1, ω, 2 3 , 2 3 ω, 2 3 , 2 3 ω⎬ .
⎪⎩ ⎪⎭
∴ its degree over Q is 6 .

c) Check that the splitting field is Q( i ) over Q , and degree


is 4 .

Q7) Use Eisenstein’s criterion to check that x 4 − 2 is irreducible over Q .

Over Q( 2 ) and Q( 2 , i) it factors as ( x 2 − 2 ) ( x 2 + 2 ) .


Over Q(α, i) it factors as ( x + α) ( x − α) ( x + iα) (x − iα) .

Q9) Any element of K is of the form a + bα .


If char F ≠ 2 , (a + bα ) 2 ∈ F iff 2abα = 0 iff a = 0 or b = 0 . Thus, the
elements are F ∪ {bα | b ∈ F} .
If char F = 2, (a + bα ) 2 ∈ F ∀a , b ∈ F .

50
Q10) As shown in the case of a biquadratic extension, show that Galois Theory
G (K / F) = H 1 × H 2 × H 3 , where H i = {I, σ i } ,
σ1 : K → K : σ1 ( 2 ) = − 2 , σ1 ( 3 ) = 3 , σ1 ( 5 ) = 5
σ 2 : K → K : σ 2 ( 3 ) = − 3, σ 2 ( 2 ) = 2 , σ 2 ( 5 ) = 5
σ 3 : K → K : σ 3 ( 5 ) = − 5, σ 3 ( 2 ) = 2 , σ 3 ( 3 ) = 3
{
Also [K : F] = 8 , a basis being 1, 2 , 3 , 6 , 5 , 10 , 15 , }
30 .
Thus, | G (K / F) | = [K : F] , so that K / F is Galois.

Q13) Let K = F(α 1 , K , α n ) . Let f i be the irreducible polynomial of α i


n
over F and f = ∏ f i . Let L ⊇ K be the splitting field of f over F .
i
Then G (L / F) acts faithfully on all the roots of f by Proposition
(1.14). Therefore, G (L / F) is finite. Also, G (K / F) ⊆ G (L / F) , so that
G (K / F) is finite.

Solutions to selected question of Section 5, Exercises, Chapter 14

Q2) Since K / F is normal, | G (K / F) | = [K : F] . Also G (K / F) ≤ S n .


Hence the result.

Q3) Let | G | = n . By Cayley’s theorem, G ≤ S n . Take


F = Q( x 1 , x 2 , K , x n ) . For each σ ∈ S n , define
φ : F → F : ϕ( x i ) = x σ( i ) and extend this to define a Q -automorphism
of F . So, S n ≤ G (F / Q) . The fixed field K = F Sn is such that F / K is
Galois with G (F / K ) = S n . Then by the Main Theorem of Galois
theory G will correspond to an intermediate field L(i.e., K ⊆ L ⊆ F) .
Thus, G (F / L) = G .

Q12) a) αp − α − a = 0 .
Now (α + 1) p = α p + 1 . ∴ f (α + 1) = 0 .

b) Let L be the splitting field of f over F . Then L / F is finite


separable. So L = F(α) . Thus, G (L / F) is of order p , and
hence is cyclic.

51
UNIT 11 APPLICATIONS OF FINITE
FIELDS

Structure Page No.


11.1 Introduction 52
Objectives
11.2 Block Designs 52
11.3 Error Correcting Codes 61
11.4 Linear Feedback Shift Registers 68
11.5 Summary 74
11.6 Solutions/Answers 74

11.1 INTRODUCTION

The goal of this unit is to share with you some applications of the algebra you
have studied so far. We will see how to apply our knowledge of finite fields in
areas like designs, error correcting codes and cryptogaphy. In Sec. 11.2, we will
introduce you to combinatorial designs. In Sec. 11.3, we will introduce you to
error correcting codes. In the last section, Sec. 11.4, we will discuss LFSRs
which are used to generate pseudorandom numbers and are useful in
cryptography.

Objectives
After studying this unit you should be able to:
• define a design, and give examples of designs;
• define and give examples of a generator matrix and a parity check matrix of an
error correcting code;
• explain what LFSRs are and how they are useful in cryptography.

11.2 BLOCK DESIGNS

In this section we construct some interesting mathematical objects using finite


fields called block designs. These objects are closely related to other
interesting combinatorial objects called Latin Squares. The study of block
designs and latin squares goes back to Euler with contributions by Steiner,
Kirkmann and others.

In the early part of the 20th century, R. A. Fisher used block designs in design of
agricultural experiments. Since then statisticians and engineers have applied
block designs in studying many industrial processes. (If you want to know more
about the applications of block designs, you may consult books on design of
experiments.) Indian Mathematicians like R.C. Bose and S. S. Shrikande have
also made important contributions to the study of block designs and Latin
squares.

We will introduce the concept of design through an example. Suppose a


university has to send a team of four players to represent it in a rowing
competition and there are 16 people who satisfy certain conditions including
52 fitness, weight, etc. What is the best way of selecting the team?
If it is an individual event like swimming, we can rank the students according to Applications of Finite Fields
the performance and select the first four fastest swimmers. On the other hand, if
it is a team event like rowing, we have to see also how good the coordination is
between the members of the team. One way of testing this will be to form all
possible teams with four members and rank them according to the time and
choose the team with the best time.

Let us call the set of aspirants X. Then, we have to take all the possible subsets of
size four from the set X, rank the performances of the teams corresponding to the
sets and select the best team. But, this is not practical if X is large. For example,
if |X| = 16, i.e. there are 16 aspirants, we will need C(16, 4) = 12!4!
16!
= 1820
trials. Note that we denote the
number of ways of
So, we are forced to choose the teams in such a way that the number of trials is choosing r elements from
manageable while ensuring that all the aspirants get a ‘fair’ chance. To ensure a set of n elements by
fairness, we have to make sure that every aspirant takes part in the same number C(n, r). Many nauthors
 use
of trials, i.e., she is a part of the same number of teams. Also, it will be unfair to a the notation r .
good rower to be always grouped with bad partners because it can affect her
chances of selection. So, every rower has to be grouped with every other player
equally often.

Let us formulate the problem in mathematical terms. In the set X of size 16, we
have to find a collection B of subsets of size four of X such that:
1) There is a number r such that each x ∈ X is in exactly r subsets of B.

2) There is a number λ such that every two element subset of X is a subset of


exactly λ subsets of B.
Note that, in the context of our problem of selecting a rowing team, the first
condition ensures that each aspirant takes part in an equal number of trials. The
second condition ensures that any two aspirants are paired together in an equal
number of trials. Our search for such a configuration of sets and subsets leads to
the concept of design.
Definition 1: Suppose t, ν, k and λ are integers with ν ≥ k ≥ t ≥ 1 and λ ≥ 1. A
simple t-design on ν points with block-size k and index λ is an ordered pair
(X, B), where:
1) X is a set with ν elements whose elements are called points.

2) B is a set of subsets of X. The elements of B are called blocks, and each


block has k elements.

3) Any subset of X with t elements is contained in exactly λ of the subsets of


B, where λ ∈ N. We usually denote the cardinality of B by b.
We call such a design t-(ν, k, λ ) design.

Those designs where ν = k are called complete designs because B = X for all
B ∈ B. If ν < k we call our design an incomplete design. When t = 2, any pair
of elements in B is in exactly λ blocks. We call such a design a balanced design.
So, in the special case where t = 2 and ν < k, (X, B) is called a balanced,
incomplete, block design.

Also, for the third condition in Definition 1 to hold true, we should have k ≥ t. 53
Study Guide-II In the definition the word ‘simple’ means that we do not allow repetition of
blocks. In more general definition of designs, repetition of blocks are allowed, i.e.
B is a collection rather than a set.
In the case of our problem of designing trials for the selection of rowing team, we
need a design with ν = 16, t = 2 and k = 4. Of course, we would like b to be as
small as possible so that the number of trials is the minimum possible.
To quote [6], a t-(ν, k, λ ) design is ‘... a collection of committees chosen out of ν
people, each committee containing k persons, and such that any t persons serve
together on exactly λ committees.’
Remark 1: We have X = ∪B∈B B. Since B ⊂ X ∀B ∈ B , ∪B∈B B ⊂ X. So, we
need to show that X ⊂ ∪B∈B B. Let x ∈ X. Consider the set
T = {x1 = x, x2 , x3 , . . . , xt }
where x2 , x3 , . . ., xt are arbitrary elements of X. By definition, T ⊂ B for at least
one B ∈ B since we have assumed that λ ≥ 1. In particular, x1 = x will be in this
B. So, X ⊂ ∪B∈B B. Thus, X = ∪B∈B B.
Let us look at a simple example of a design.
Example 1: Create a 2-(9, 3, 1) design on X = {1, 2, 3, 4, 5, 6, 7, 8, 9}.
Solution: Let
B = {{1, 2, 3}, {4, 5, 6}, {7, 8, 9}, {1, 4, 7}, {2, 5, 8}, {3, 6, 9},
(1)
{1, 5, 9}, {2, 6, 7}, {3, 4, 8}, {1, 6, 8}, {2, 4, 9}, {3, 5, 7}}.
Here X has 9 elements so ν = 9. Further every subset in B has three elements so
k = 3.
Notice that any pair of elements of X are in exactly one subset in B. For example
the elements 1 and 2 are there only in the first subset {1, 2, 3} and in no other
subset. If you like you can check this for every pair of elements in X. (There are
only C(7, 2) = 21 pairs in all!) So, t = 2 and λ = 1. This is a 2-(7,3,1) design.
∗∗∗
One important class of designs are the projective planes. To construct these
designs, we fix a vector space V of dimension 3 over a finite field Fq and take X
to be set of all one dimensional subspaces of V. The blocks correspond to two
dimensional subspaces of V. Then, we get a 2-(q2 + q + 1, q + 1, 1) design. A
detailed discussion of these designs is beyond the scope of this unit. In the next
example, we discuss a particular case of projective planes where q = 2. The
block design we get is called called the Fano plane.
Example 2: Construct the Fano plane.
Solution: Let F be a finite field with 2 elements. Let V = F3 and X be the set
of all one dimensional subspaces of V. In general any vector space of dimension
n over the finite field Fq has qn elements. So, the underlying set of any one
dimensional vector space over F2 has exactly two elements, zero element being
one of them. For example {(0, 0, 0), (1, 0, 0)} is a vector space of dimension one
over F2 . Let us now list all the vector spaces of dimension 1 over F2 , giving each
of them a name for easy reference. We have,

A = {(0, 0, 0), (0, 1, 0)}, B = {(0, 0, 0), (1, 0, 1)},⎪


C = {(0, 0, 0), (0, 0, 1)}, D = {(0, 0, 0), (1, 1, 1), ⎬
(2)
E = {(0, 0, 0), (0, 1, 1)}, F = {(0, 0, 0), (1, 0, 0)} ⎪



54 G = {(0, 0, 0), (1, 1, 0)}
So, Applications of Finite Fields

X = {A, B, C, D, E, F, G}.

Let us denote the one-dimensional subspace generated by the vector (a, b, c),
(a, b, c) = 0, by [a : b : c]. For example we denote the one-dimensional vector
space {(0, 0, 0), (0, 0, 1)} by [0 : 0 : 1].

As you can see in Eqn. (2), there are exactly 7 one-dimensional subspaces,
namely, [1, 0, 0], [0 : 1 : 0], [0 : 0 : 1], [1 : 1 : 0], [0 : 1 : 1], [1 : 0 : 1] and [1 : 1 : 1].
Note that every two-dimensional vector space over F has four elements and there
are seven two-dimensional subspaces of F3 . They are

{(0, 0, 0), (0, 0, 1), (1, 0, 0), (1, 0, 1)}, {(0, 0, 0), (1, 0, 1), (1, 1, 1), (0, 1, 0)}
{(0, 0, 0), (0, 1, 0), (0, 1, 1), (0, 0, 1)}, {(0, 0, 0), (1, 0, 0), (1, 1, 0), (0, 1, 0)}
{(0, 0, 0), (1, 0, 1), (1, 1, 0), (0, 1, 1)}, {(0, 0, 0), (1, 1, 1), (1, 1, 0), (0, 0, 1)},
and {(0, 0, 0), (0, 1, 1), (1, 1, 1), (1, 0, 0)}.

Let us now define the blocks of our design as follows: For each two-dimensional
subspace W of V, we define a block BW as follows:

BW = {S ∈ X|S ∩ W = {(0, 0, 0)}}

For example, if we take W0 = {(0, 0, 0), (0, 0, 1), (1, 0, 0), (1, 0, 1)}, then
S ∩ W = (0, 0, 0) for the one dimensional subspaces

[0 : 0 : 1] = {(0, 0, 0), (0, 0, 1)} = C,


[1 : 0 : 0] = {(0, 0, 0), (1, 0, 0)} = F,
[1 : 0 : 1] = {(0, 0, 0), (1, 0, 1)} = B.

So,

BW0 = {{(0, 0, 0), (0, 0, 1)}, {(0.0, 0), (1, 0, 0)}, {(0, 0, 0), (1, 0, 1)}} (3)
= {[0 : 0 : 1], [1 : 0 : 0], [1 : 0 : 1]} = {B, C, F} (4)

We can represent the system as in Fig. 1. The points in the figure represent the
points in the design and the lines represent the blocks in the design. Note that the

[0 : 0 : 1] C

[0 : 1 : 1] E F [1 : 0 : 0]
G[1 : 1 : 0]

D
[0 : 1 : 0] A B[1 : 0 : 1]
[1 : 1 : 1]

Fig. 1: Fano plane

block corresponding to each two-dimensional subspace of V has three non-zero


one-dimensional subspaces. In the figure each line, including the curved line
FDE, corresponds to the block associated with a two-dimensional subspace of V.
For example the line CFB corresponds to the block BW0 in Eqn. (3). 55
Study Guide-II We have shown the three non-zero points(one-dimensional subspaces) in each of
the blocks. For example {A, E, C} is the block associated with the
two-dimensional subspace of V spanned by the vectors (0, 0, 1) and (0, 1, 1). So,
k = 3 in this example.
Note that any two distinct points are in a unique block. If [a : b : c] and [a
: b
: c
]
are two distinct points, (a, b, c) and (a
, b
, c
) are linearly independent. So, they
determine a unique two-dimensional subspace W of V. It follows by the
definition of BW that [a : b : c] and [a
: b
: c
] ∈ BW . So, t = 2 and λ = 1 in this
case.
So, the blocks are {A, E, C}, {C, F, B} {A, D, B}, {A, G, F}, {E, G, B}, {C, G, D}
and {E, D, F}.
∗∗∗
You may have noticed that λ = 1 in Example 1, Example 2 and Example 3. Such
special designs are called steiner systems.
Definition 2: A steiner system is a t-design with λ = 1. We denote a steiner
system by S(t, k, ν) instead of t-(ν, k, 1).
In other words a steiner system S(t, k, ν) is a design in which each subset of X
with t elements is contained in exactly one of the subsets in B. In what follows,
by S(t, k, ν), we mean a steiner system with parameters t, k and ν. Fano plane is
an example of steiner system; it is an S(2, 3, 7).
We now construct another design using finite fields. Notice that this is also a
steiner system.
Example 3: Let X be the additive group of the vector space Fn2 . A subset
{a, b, c, d} ⊂ X is a block if a + b + c + d = 0 in Fn2 . Check that this gives us a
S (3, 4, 2n ).
Solution: For this we have to check that any subset of Fn2 with three elements
is contained in a unique block.
Suppose T = {a, b, c} ⊂ X is a set with 3 elements. Let d = a + b + c. First, we
claim that d  ∈{a, b, c}. Suppose d ∈ {a, b, c}. We can assume without loss of
generality that d = a. Then, b + c = 0 or b = c since x = −x for all x ∈ Fn2 . This
contradicts the fact that T is a set with distinct elements.
If we choose d = a + b + c, then a + b + c + d = 2(a + b + c) = 0 in Fn2 . So,
B = {a, b, c, d} is a block and T is contained in this block. If T is contained in
another block B
= {a, b, c, d
}, we must have d
= a + b + c = d. So, B = B
.
∗∗∗
A trivial way of constructing a design (X, B) is to include all possible subsets
B ⊂ X with |B| = k in B. We discuss the properties of this design in the next
example.
Example 4: Let X be a set with |X| = ν and k ≤ ν. Suppose we take B to be
set of all subsets of X of cardinality k. Check that for each t ≤ k, (X, B) is a
t-(ν, k, λ ) design for some value of λ .
Solution: To show that (X, B) is a t-(ν, k, λ ) design, we have to show that any
subset of t elements of X is contained in the same number of subsets in B.
Let T ⊂ X be any subset of cardinality t. We can add k − t elements to T to get a
56 set of cardinality k which contains T. We have to choose these k − t elements
from the set X \ T. Since X \ T has ν − t elements, we can choose the k − t Applications of Finite Fields
elements out of the ν − t elements in

C(ν − t, k − t) = C(ν − t, (ν − t) − (k − t)) = C(ν − t, ν − k) (5) Recall that


C(n, r) = C(n, n − r).
ways. Thus, T is contained in exactly C(ν − t, ν − k) blocks. So, we get a design
with λ = C(ν − t, ν − k). This is a trivial design.
∗∗∗
Let us now look at an example of a trivial design.
Example 5: Construct a trivial design with X = {1, 2, 3, 4, 5, 6} and k = 4. What
are the possible values of t? Find the value of λ corresponding to the values of t.

Solution: In this example ν = 6, k = 4. Let

B = {S ⊂ X||S| = 4},

be the set of all subsets of X with four elements. There are C(6, 4) = 15 subsets
of X of size 4. We can take any value of t, t ≤ 4. Our λ will depend on the value
of t. Suppose t = 4. From Eqn. (5), we get

λ = C(6 − 4, 6 − 4) = 1

so, any set of 4 elements in contained in exactly one set of 4 elements, which is a
self evident statement.

Let us take t = 3. In this case,

λ = C(6 − 3, 6 − 4) = C(3, 2) = 3.

So, any subset of 3 elements of X will be contained in exactly 3 subsets in B. So,


(X, B) is also a 3-(6,4,3) trivial design.

Again, if we take t = 2, we get

λ = C(6 − 2, 6 − 4) = C(4, 2) = 6.

So, any pair of elements in X are in exactly six subsets in B. So, (X, B) is a
2-(6,4,6) trivial design.
∗∗∗
Here is an exercise to check your understanding of the discussion so far.

E1) Let X = {1, 2, 3, 4, 5, 6, 7} and B be the set of subsets of X with five EXERCISES
elements. Find the value of λ for (X, B) considered as a 4 design and a 3
design.

We will now see some methods for constructing new designs from existing
designs. We begin with a simple construction.
Theorem 1: Suppose X is a t-(ν, k, λ ) design. Let 0 ≤ i ≤ t and suppose I ⊂ X,
|I| = i for some t-(ν, k, λ ) design. The number of blocks B such that I ⊂ B is
C(ν − i, t − i)
λi = λ (6)
C(k − i, t − i)
In other words, every t-(ν, k, λ ) is also also an i-(ν, k, λi ) design for i ≤ t. 57
Study Guide-II Proof: Let us fix a set I with i elements and suppose there are λi sets B that
contain I. So, λi may be depend on I. We will see later in the proof that this is not
so.

We use a standard technique in combinatorics. We count the number of elements


in a set in two different ways and equate them. Consider the set

{ (T, B)| I ⊂ T, T ⊂ B, |T| = t, } (7)

Let B be a block containing I. Other than the elements of I, there are k − i


elements in B. Out of these k − i elements of B, we can choose any t − i elements,
add them to the set I and get a set T with T ⊂ B, and |T| = t. Since there are λi
choices for B and C(k − i, t − i) ways of adding t − i elements of B to I to get a t
element subset of B, the number of elements of the set in Eqn. (7), is
λi C(k − i, t − i).

On the other hand, we can add t − i elements to I to get a subset of size t in


C(ν − i, t − i) ways. Each such subset is contained in exactly λ blocks. So, the
number of elements in the set in Eqn. (7) is C(ν − i, t − i)λ . Equating these two
values we get

λi C(k − i, t − i) = C(ν − i, t − i)λ


C(ν − i, t − i)
i.e. λi = λ . (8)
C(k − i, t − i)

Note that, the expression for λi depends only on λ , ν, k, t and i and not on the
particular set I ⊂ X we have chosen. So, Eqn. (8) is true for any I ⊂ X of size
i. 
There is an interesting corollary that follows from the result just proved. This
corollary relates various parameters associated with a design. So, it helps us
sometimes in deciding whether it is possible to construct a design with a given set
of parameters.
Corollary 1: 1) Given a t-(ν, k, λ ) design, the parameters b, ν, λ and k satisfy
the relation
C(ν, t)
b=λ (9)
C(k, t)

2) Every element of X will be in precisely


C(ν − 1, t − 1)
λ1 = λ (10)
C(k − 1, t − 1)
of the blocks.

Proof: 1) Take i = 0 in Eqn. (8). We have λ0 = b since the empty set is a


subset of every set.

2) Take i = 1 in Eqn. (8) to get the result.




Remark 2: If we take t = 2 in Eqn. (10), i.e. the design is a balanced, incomplete


block design. We have
ν −1
58 λ1 = λ . (11)
k−1
In this case λ1 is called the replication number and usually denoted by r. So, in Applications of Finite Fields
the case of a balanced, incomplete, block design, the replication number r
satisfies the condition
r(k − 1) = λ (ν − 1) (12)
Further, in this case, if we take t = 2 in Eqn. (9), we get
ν(ν − 1)
b=λ
k(k − 1)
ν
= r, from Eqn. (12)
k
i.e. bk = rν (13)
Let us look at an application of Eqn. (9).
Example 6: Note that, to solve the problem of conducting trials for selecting a
rowing team, we need to construct a block design with ν = 16, k = 4, t = 2,
λ = 1. Use Eqn. (9) to find b in the situation where we want to conduct trials for
the rowing team.

Solution: From Eqn. (9), we have


C(ν, t) C(16, 2) 120
b=λ =λ =λ = 20λ ,
C(k, t) C(4, 2) 6
Thus, 20 | b. So, we will need at least 20 trials. Also, if λ = 1 for the putative
design, b = 20.

Of course, the fact that the parameter values satisfy Eqn. (9) doesn’t guarantee
that a 2-(16, 4, 1) design exists. Eqn. (9) is only a necessary condition. So, we
now describe a design with b = 20 without showing its construction.

We can take X = {1, 2, 3, . . . , 16} and


B = {{1, 2, 3, 4}, {5, 6, 7, 8}, {9, 10, 11, 12}, {13, 14, 15, 16}, {1, 5, 9, 13},
{2, 6, 10, 14}, {3, 7, 11, 15}, {4, 8, 12, 16}, {1, 6, 11, 16}, {2, 5, 12, 15},
{3, 8, 9, 14}, {4, 7, 10, 13}, {1, 7, 12, 14}, {2, 8, 11, 13}, {3, 5, 10, 16},
{4, 6, 9, 15}, {1, 8, 10, 15}, {2, 7, 9, 16}, {3, 6, 12, 13}, {4, 5, 11, 14}}
Further, note that each element of X is in exactly five subsets in B, i.e. λ = 1 as
expected. Also, every pair of elements of X is in exactly one subset of X, so any
two players are paired exactly once. In conclusion, we can select the rowing team
with just 20 trials!
∗∗∗
Here are some exercises for you to try and check your understanding.

E2) Suppose D = (X, B) is t-(ν, k, λ ) design and let 0 ≤ j ≤ t. If J ⊂ X, |J| = j, EXERCISES


let us denote by λ (j) the number of blocks of B ∈ B such that B ∩ J = φ .
Then, we have
C(ν − j, k)
λ (j) = λ (14)
C(ν − t, k − t)
(Hint: Count the number of pairs (J, B) with J ∩ B = φ in two ways to get
λ j C(ν, j) = bC(ν − k, j). Use Eqn. (9) to eliminate b and regroup the
terms.)

E3) Show that, if there is an S(3, 6, ν), then ν ≡ 2 (mod 20) or ν ≡ 6


(mod 20). 59
Study Guide-II E4) Check whether it is possible to have balanced, incomplete, block designs
with the sets of parameters given below:

No. ν r k b λ
1 7 3 3 7 1
2 12 3 3 4 1

In the next proposition, we state some methods of constructing new designs from
existing designs.
Proposition 1: Let D = (X, B) be a t-(ν, k, λ ) design.
1) Let
B
= {B ⊂ X||B| = k, B ∈ B} .
In other words, choose all the subsets of X with k elements which are not in
B as blocks. Then (X, B
) is a t-(ν, k, C(ν − t, k − t) − λ ) design. This is
called the complementary design of D.

2) Fix a set I ⊂ X with |I| = i, i < t. Let


X
= X \ I, B
= { B \ I| B ∈ B, B ⊃ I} .
Then, (X
, B
) is a (t − i)-(ν − i, k − i, λ ) design. This is called the derived
design of D.

3) Let
B
= { X \ B| B ∈ B} .
i.e., choose as blocks the complements of the blocks of B, i.e. X \ B,
B ∈ B. Then, (X, B
) is a t-(ν, ν − k, λ
) design where λ
= λ C(ν−k,t)
C(k,t) .
This is called the supplementary design of D.

Proof: 1) All we need to prove is that every set T, T ⊂ X, of size t is


contained in C(ν − t, k − t) − λ subsets of X which are of size k and are not
in B. We can choose a set of size k − t which is disjoint from T in
C(ν − t, k − t) ways. Taking the union of this set with T, we get a set of size
k. So, there are C(ν − t, t − k) subsets of size k which contain T. Of these,
λ of the sets are in B , so the remaining sets, which are C(ν − t, k − t) − λ
in number, are in B
.

2) Let us take a set S, S ⊂ X


, with t − i elements. We need to show that the
number of subsets in B
that contain S is λ . Take T = S ∪ I. Then, T ⊂ X
and |T| = t, so T is contained in exactly λ of the sets in B. Further,
B → B ∪ I gives a one-to-one correspondence between the sets in B
that
contain S and the sets in B that contain T. This completes the proof.

3) We have to show that, any subset of t elements of X are in the same number
of blocks in B
. Let T be a subset with t elements. Note that, the number of
blocks B
∈ B
that contain T is the same as the number of blocks in B
with B ∩ T = φ . This is because, if T ⊂ B
, B
∈ B
then, T ∩ B = φ for the
block B = X \ B
where B ∈ B by definition. By Eqn. (14), the number of
subsets of blocks that do not intersect T is
C(ν − t, k)
λ .
60 C(ν − t, k − t)
Check that, Applications of Finite Fields
C(ν − t, k) C(ν − k, t)
λ =λ 
C(ν − t, k − t) C(k, t)
Let us look at an example that illustrates the use of Proposition 1.
Example 7: Consider the Fano plane, which is a S(2, 3, 7) steiner system.
1) Construct its complementary design.

2) Construct the derived design corresponding to I = {G}.

3) Construct the supplementary design.


Solution:
1) Its complementary design is as follows: We have X = {A, B, C, D, E, F, G}.
Note that, there are seven subsets of order 3 and a set of order seven has
C(7, 3) = 35 sets of order three. So, the complementary design will have
35 − 7 = 28 blocks, i.e. b = 28. The value of λ is
C(7 − 2, 7 − 3) − 1 = C(5, 2) − 1 = 9. So, the complementary design is a
2-(7, 3, 9) design.

2) Let us now construct a derived design using I = {G}. We have


X = {A, B, C, D, E, F, G}. There are only three blocks in B that contain G,
namely {E, G, B}, {A, G, F} and {C, G, D}. Removing G from these blocks,
we get three blocks {E, B}, {A, F} and {C, D}. So, n = 6, b = 3, k = 2,
t = 1 and λ = 1. So, we have a S(1, 6, 2) steiner system.

3) Let us now construct the supplementary design corresponding to the Fano


plane. In a supplementary design, X remains the same. We replace the
blocks by their complements. So, the blocks in this case are
X \ {A, E, C} = {B, D, F, G}, X \ {C, F, B} = {A, D, E, G},
X \ {A, D, B} = {C, E, F, G}, X \ {A, G, F} = {B, C, D, E},
X \ {E, G, B} = {A, C, D, F}, X \ {C, G, D} = {A, B, E, F} and
X \ {E, D, F} = {A, B, C, G}. The value of λ for this design is
1 · C(7−3,2)
C(3,2) = 2. So, this is a 2-(7, 4, 2) design.
∗∗∗
We close the section here. In the next section, we discuss error correcting codes
which play an important role in digital communication.

11.3 ERROR CORRECTING CODES

In this section, we will discuss error correcting codes, often called simply as
codes. We can think of codes as a means of expressing the messages in a form
that is suitable for our method of communication. We usually use various
symbols with each symbol standing for a particular message. For example traffic
signals convey three different messages using three different colours, Green,
Orange and Red. Other examples are the traffic signs we see on the roads.(See
Fig. 2.) Instead of pictures, we use words also to convey specific messages. They
are called codewords. For example, ‘MAYDAY’ is the standard distress signal
sent by an aircraft or a ship which needs to be rescued from a dangerous situation.

In this unit we are interested only in digital communication. We would like to


communicate our information efficiently, securely and without errors. We design 61
Study Guide-II

Fig. 2: Traffic Signs.

and use the symbols in such a way that we can achieve these goals. In this
section, we will focus on codes that help us to communicate digitally with
minimum possible errors. Fig. 3 shows a communication channel without error

Noise

M M
Sender Receiver

Fig. 3: Channel without error correction.

correction facilities. Here, the message M sent by the sender gets distorted into
M
due to some disturbance in the media. For example, signals from satellites are
transmitted using radio waves and this could be affected by Sun’s radiation. The
recipient may not be able to make out whether there is a mistake in the message.
On the other hand, we can see in Fig. 4 a channel with error correction facilities.
In this, instead of sending the message M itself, the encoder replaces the message
M by the corresponding codeword C. (We will clarify what a codeword is in the
context of error correction codes later.) The noise in the channel distorts C to C
.
The codewords are so designed that the decoder can recover the correct code
word C from the distorted codeword C
subject to certain conditions. The decoder
corrects the error and finds the correct transmitted codeword and recovers the
original message M. In this section, we are interested in such encoding processes

Noise

M C C M
Sender Encoder Decoder Recipient

Fig. 4: Channel with error correction.

62 that will enable us to correct the errors in the transmitted message.


We will confine ourselves to binary, linear, codes in this unit. Let us begin by Applications of Finite Fields
defining these objects.
Definition 3: A linear, binary code C of length n and dimension k is a
k-dimensional subspace of Fn2 . The elements of C are called codewords of C .
We also say that C is an [n,k]-code.

We often write the codewords of a code C of length n in the form of a string


instead of an n-tuple. In other words, instead writing a codeword as
(a1 , a2 , . . . , an ), ai ∈ {0, 1}, we will write it as a1 a2 · · · an . In the remaining part of
this section, we will denote the field F2 by F.
Remark 3: We call the codes in Definition 3 binary because we use binary
strings as codewords. We call our codes linear because the codewords constitute
linear spaces over F2 .
Let us now look at a simple example of a code.
Example 8: Suppose we just want to transmit the 128 characters of the ASCII
table. (You can see an ASCII table in the first block of MMT-001.) We represent
the characters using the numbers 0 to 127. We can represent any number between
0 and 127 as a seven digit binary number. So, we can represent every character
by a 7-tuple or a binary string of length seven. For example, the ASCII value of
B is 66, which is 1000010 in binary.

Suppose we transmit the ASCII value of B in binary and the last bit changes from
1 to 0 because of the noise in the transmitting medium. The recipient will receive
A instead of B. To handle such errors, the recipient should be able to detect the
error and take corrective measures. Explain how adding an extra bit will help in
detecting errors. Also, explain why the set of codewords in your message forms a
vector subspace of Fn for some n, i.e. the code is a linear code.

Solution: We count the number of 1’s in the message. If the number of 1s is


odd, we add a 1 at the end; if the number of 1s is even, we add a zero at the end.
In particular, if we want to send the character ‘A’, instead of sending 1000001,
we send 10000010. We call the extra bit the parity check bit.

So, we send eight bits instead of seven bits and the recipient checks whether there
are even number of 1’s. If there isn’t, she will know that an error has occurred
She can then ask the sender to send that part of the message again.

When we added the parity bit, we modified the message by adding some bits
which are not part of the message. We call the extra bits the redundant bits. The
process of adding extra bits for error correction is called encoding the message.
Once the recipient receives, say, 10000010, she can extract the message ‘A’ from
this codeword by discarding the last bit of the codeword after checking for error.
This process is called decoding the message.

Note that, if we add all the eight bits of a codeword, treating them as elements of
F, we will get zero since any codeword always has even number of 1’s.
Conversely, if the sum of the digits is zero, the number of 1s has to be even. So,

C = x1 x2 · · · x8 ∈ F8 |x1 + x2 + · · · + x8 = 0
Note that, C is the kernel of the linear map F8 → F, given by
x1 x2 · · · x8 → x1 + x2 + · · · + x8 . 63
Study Guide-II We leave it to you to check, using the rank-nullity theorem, that C is a seven
dimensional subspace of F8 . So, C is an [8, 7]-code.
∗∗∗
Let us look at another example of a code in which we can correct some of the
errors instead of merely detecting the errors.
Example 9: Suppose we want to transmit the 128 characters in the ASCII table.
Explain how to construct a code that uses repetition of the message for encoding
it. Explain why your code is a linear code. Also, explain how you can correct
some of the errors.

Solution: To encode the message, we send the binary equivalent of each


character thrice. For example, if we want to transmit the character ‘A’, we send
the binary string 100000110000011000001. Note that, in this case,

C = x1 x2 · · · x21 ∈ F21 |xi = xj if i ≡ j (mod 7)
In other words, the codewords are vectors in C whose first, eighth and fifteenth
coordinates are the same, second, ninth and sixteenth coordinates are the same,
etc. This is called a repetition code. Since C is a vector subspace of F21 , C is a
linear code.

We decode the messages in this code is as follows: We check the positions which
should be equal and take whichever among the two symbols, 0 and 1, that occurs
more times. For example, suppose we receive 1000001 0100001 1000001. (We
have introduced spaces for convenience.) For convenience, we split this into
chunks of seven and write the three chunks one below the other:
1 0 0 0 0 0 1
0 1 0 0 0 0 1
1 0 0 0 0 0 1
1 0 0 0 0 0 1
The fourth line in the above table gives the decoded message. We examine each
column and see which symbol occurs most frequently in that column and write
down that symbol under that column. For example, in the first column from the
left, there are two ones and one zero. So, we take the left most bit in the decoded
message to be one.

We can always correct one error using this code. For example, suppose we
receive the string 1010001 1000001 1000001 and we see that the third bit is 1
but the tenth and seventeenth bits are 0. If we assume that there is at most one
error, we infer that the error is in the third bit and correct it.

We may not be always to able to correct two errors although we can detect two
errors. Suppose we receive that string 1100001 1100001 1000001. We see that
the second, ninth are 1 and the sixteenth bit is 0. If we assume that there can be
two or more errors, there are two possibilities. The first possibility is second and
the ninth bits are correct and the sixteenth bit is wrong and therefore there is only
one error. The other possibility is that the second and ninth bits are wrong and the
sixteenth bit is correct, implying that there are two errors. So, we can infer infer
that there are errors in the message. But, we can correct the errors only if there is
only one error. We leave it to you as an exercise to check that C is a [21, 7]-code.
∗∗∗
Here are some exercises for you to check your understanding of the above
64 examples.
Applications of Finite Fields

E5) Check that, the code in Example 9 is a [21, 7]-code. EXERCISES

E6) Again, let us consider the code in Example 9. Suppose you receive the
message 1000111 1000111 1000101. Assuming that the number of errors
is not more than one, find the message.

Note that, since a linear code is a linear subspace of Fn , once we know a basis for
the code, we can reconstruct all the codewords of the code. So, for each linear
code we associate a matrix whose rows are the basis elements of that code. We
call this matrix a generator matrix of the linear code. Let us look at an example
to understand this.
Example 10: Consider the code
C = {0000, 0011, 0101, 0110, 1001, 1010, 1100, 1111}
Find the generator matrix of the code.

Solution: Since the code has 23 = 8 elements, we know that the dimension of
the linear code as a subspace of Fn is 3. So, to find a generator matrix, we have to
find three linearly independent vectors. We can do this by trial and error. We
select any three elements and form a 3 × 4 matrix and check whether it has rank
three. If the matrix has rank three, the row vectors are linearly independent and
so form a basis for the code C .

For example, consider the following set of three elements.


{0011, 0101, 1001}
⎡ this in the⎤ form of a 3 × 4 matrix
. We can write
0 0 1 1
G = ⎣0 1 0 1⎦ .
1 0 0 1
We can see that the determinant of the matrix formed by the first three columns is
1 and hence non-zero. So, the rows of G are linearly independent and G is a
generator matrix for C .
∗∗∗
Here is the formal definition of a generator matrix of a linear code.
Definition 4: Let C be an [n, k]-code. A k × n matrix with coefficients in F is a
generator matrix for C if its rows form a basis for C .
Let us look at some examples of generator matrices. Before we proceed further,
some remarks about generator matrices are in order.
Remark 4: 1) Suppose C is an [n, k]-code. A k × n matrix over F is a
generator matrix for the code if every row of the matrix is a codeword and
the rank of the matrix is k.(Why?)

2) Note that, if we rearrange the rows of a generator matrix of a code, they still
form a basis. More generally, if we perform any of the following row
operations, the resulting matrix will still be a generator matrix.
a) Multiply a row by a nonzero scalar.
b) Multiply a row by a scalar and add it to another row.
c) Interchange rows. 65
Study Guide-II So, a generator matrix of a code is not unique. However, we can use row
reduction to reduce a generator matrix to reduced row echelon form and this
is unique.

3) If the generator matrix of a code is of the form [Ik , P] where Ik is the k × k


identity matrix and P is a k × (n − k) matrix, then we say that the generator
matrix is in systematic form.
Example 11: Construct the generator matix for the code in Example 8.

Solution: We saw that, in this case


C = {x1 + x2 + · · · x8 |x1 + x2 + · · · + x8 = 0}
Consider the⎡ matrix ⎤
1 0 0 0 0 0 0 1
⎢0 1 0 0 0 0 0 1⎥
⎢ ⎥
⎢0 0 1 0 0 0 0 1⎥
⎢ ⎥
G=⎢ ⎢0 0 0 1 0 0 0 1⎥

⎢0 0 0 0 1 0 0 1⎥
⎢ ⎥
⎣0 0 0 0 0 1 0 1⎦
0 0 0 0 0 0 1 1
We claim that G is the generator matrix for C . By part two of Remark 4 we have
to check that each row of the matrix is a codeword and the rank of the matrix is 7.
Recall that a vector in F8 is a valid code word if and only if it has even number of
ones. Since each row contains exactly two ones, each row is a valid codeword.
Further, note that the first 7 columns form a 7 × 7 identity matrix and so the rank
of G is 7. So, G is indeed a generator matrix for C . Note that, G is in systematic
form.

We use a generator matrix for encoding the messages, in this case the ASCII
codes. For example, the ASCII code of ‘B’ is 1000001 in binary. To encode this,
we multiply the 1000001 by the matrix G to get the encoded vector:
⎡ ⎤
1 0 0 0 0 0 0 1
⎢0 1 0 0 0 0 0 1⎥
⎢ ⎥
⎢0 0 1 0 0 0 0 1⎥
⎢ ⎥
1000001 ⎢ ⎢0 0 0 1 0 0 0 1⎥ = 10000010

⎢0 0 0 0 1 0 0 1⎥
⎢ ⎥
⎣0 0 0 0 0 1 0 1⎦
0 0 0 0 0 0 1 1
Note that, first 7 bits of the codewords consists of the original message and the
last bit is the redundant bit we use for parity check. This is true for all the [n, k]
codes for which the generator matrix is in systematic form. The first k bits of the
codeword will consist of the message bits and the remaining n − k bits will be
redundant bits for error correction.
∗∗∗
Here is an exercise for you to check your understanding.

EXERCISES E7) Check that, the generator matrix for the code in Example 9 is [I7 , I7 , I7 ].

For decoding, we first need to know whether we received a valid codeword or not.
One way of checking this is to compare it with all the codewords in the code and
66 see if the message matches any of them. However, this is time consuming. We
can do this quickly by using the parity check matrix of the code. We will discuss Applications of Finite Fields
this in the next example.
Example 12: Write down the parity check matrix for Example 8 again.

Solution: We know that a vector x is a valid codeword if and only if the sum of
all the bits is one. Note that, the sum of all bits in a codeword is simply the dot
product of the codeword with the vector (1, 1, 1, 1, 1, 1, 1, 1). So, if we write
 
H= 1 1 1 1 1 1 1

then C = x ∈ F8 | xHt = 0 . In other words, the matrix H checks whether the
parity of the number of 1s in the codeword is correct. We call H the parity check
matrix of the code C .
∗∗∗
Here is the formal definition of parity check matrix.
Definition 5: Let C be an [n, k]-code. An (n − k) × n matrix H is called a parity
check matrix for C if

C = x ∈ Fn |xHt = 0 .
We say that H is in systematic form if it is of the form [Q, In−k ] where Q is an
(n − k) × k matrix.

Note that, according to Definition 5, the parity check matrix has the property that
the kernel of the linear map Ht : Fn → Fn−k is precisely the subspace C . We
leave it to you as simple exercise to check using linear algebra that Ht is onto and
its rank is n − k.

We can find the parity check matrix from the generator matrix in systematic form
and vice versa using Proposition 2.
Proposition 2: If G = [Ik , P] is the generator matrix of, a not necessarily binary,
[n, k]-code C in systematic form, then H = [−Pt , In−k ] is a parity check matrix
for C . Conversely, if the parity check matrix of a [n, k]-code is of the form
[Q, In−k ], its generator matrix is of the form [Ik , −Qt ].

Proof: Assume that G = [Ik , P] is the generator matrix of a [n, k] linear code C .
First, we prove that, for v ∈ C , vHt = 0 for all v ∈ C . It follows that C is in the
kernel of H. Also, the rank of H is n − k, so the nullity is k by rank nullity
theorem. So, the kernel of H is precisely C .

To prove that C is in the kernel, it is enough to prove that every row of G is


annihilated by Ht . This  GH = 0. If we have a block matrix
t
 tis
 the same as  saying
A −P
[A B], then [A B]t = t . So, Ht = . We have
B In−k
 
−P
GH = [Ik , P]
t
= Ik · −P + P · In−k = −P + P = 0
In−k
and this completes the proof.

Conversely, suppose H = [Q, In−k ] is the parity check matrix of a code C .


Consider G = [Ik , −Qt ]. Then,
 t 
Q
GH = [Ik , −Q ]
t t
= Ik · Qt + −Qt · In−k = Qt − Qt = 0
In−k
So, each row of G is a valid codeword. Futher, G has rank k since first k columns
form a k × k identity matrix. So, G is a generator matrix for the code.  67
Study Guide-II Let us now look at an example to understand Proposition 2.
Example 13: Find the parity check matrix of the code in Example 9.
Solution: We already know that [I7 , I7 , I7 ] is its generator matrix in systematic
form. This is of the form [I7 , P] where P = [I7 , I7 ]. So, its parity check matrix is
[Q, I14 ], where
 
−I7
Q = −P = t
.
−I7
∗∗∗
Here is another example.
Example 14: Check that C over F2 given by
C = {000000, 010101, 101010, 111111}
is a linear code. Further, find a generator matrix and parity check matrix of the
code.
Solution: Since C is a binary code, note that, to check that C is a subspace, we
just have to check that the sum of any two elements in C is again in C . You can
check this quite easily in this case.
Note that, 101010 and 010101 generate C over F2 . So, the generator matrix in
this case
 is 
1 0 1 0 1 0
.
0 1 0 1 0 1
Here, the generator matrix is already in systematic form [I2 , P] where
 
1 0 1 0
P=
0 1 0 1
So, the parity check matrix is
⎡ ⎤
1 0 1 0 0 0
  ⎢0 1 0 1 0 0⎥
H = −Pt , I4 = ⎢
⎣1 0 0 0 1 0⎦
⎥ (15)
0 1 0 0 0 1
∗∗∗
Here is an exercise for you to check your understanding of Example 13 and
Example 14.

EXERCISES E8) Consider the code with the following generator matrix
⎡ ⎤
1 0 0 1 0 0 1 0 0
G = ⎣0 1 0 0 1 0 0 1 0⎦
0 0 1 0 0 1 0 0 1
in standard form. Find the parity check matrix of the code in standard form.

We close our discussion on error correcting codes here. We will discuss this topic
in MMTE-005, which is a full course on error correcting codes. In the next
section, we discuss Linear Feedback Shift Registers (LFSRs).

11.4 LINEAR FEEDBACK SHIFT REGISTERS

Another application of finite fields is in the construction of Linear Feedback Shift


68 Register(LFSR). Linear Feedback shift registers are hardware devices which are
used to generate pseudorandom number sequences. However, since we are Applications of Finite Fields
interested in the Mathematics behind these devices, we will describe LFSR as a
mathematical object.

You are probably wondering what pseudorandom numbers are and why we have
used the adjective ‘pseudo’. Random number sequences are a sequence of
numbers in which it is impossible to predict the next number in the sequence
even if we are given all the earlier numbers in the sequences. Such random
number sequences are generated using natural phenomena like by timing the gap
between successive pairs of radioactive decays detected by a Geiger-Müller
counter or thermal noise from a semiconductor resistor or tossing a coin.

However, such random numbers are difficult to generate in large quantities. So,
we often settle for pseudorandom sequences which are sequences generated
using some mathematical function based on an input called a seed. These
sequences satisfy all the statistical properties that are satisfied by random
numbers. However, anyone who knows the function and the seed used can
reconstruct the entire sequences. But, this is not possible to do so for random
number sequences constructed by tossing a coin or other such means.

Pseudo random numbers are useful in testing algorithms. For example, suppose
you have invented a new algorithm for sorting numbers in ascending order. You
can test how well it performs by generating data sets of numbers using a
pseudorandom generator and testing your algorithm on this data. We will see
later how pseudorandom numbers are useful in cryptography.

We begin our discussion of LFSRs with the formal definition of LFSRs.


Definition 6: Let q = pk be the power of a prime and let n be a positive integer
and c0 , c1 , . . ., cn−1 ∈ Fq . Given any n-tuple (s0 , s1 , . . . , sn−1 ) ∈ Fnq , let
s∞ = (s0 , s1 , . . . , sn−1 , . . .) denote the infinite sequence of elements of Fq
determined by the following relation:
sj+n = sj c0 + sj+1 c1 + · · · + sj+n−1 cn−1 (16)
Eqn. (16) is called a linear feedback shift register(LFSR) of order n over Fq
while we refer to s∞ as the sequence generated by Eqn. (16). We call the n-tuple
(s0 , s1 , . . . , sn−1 ) the initial state of the LFSR given by Eqn. (16) and the
polynomial xn − cn−1 xn−1 − . . . c1 x − c0 is called the characteristic polynomial
of the LFSR in Eqn. (16).

We say that the sequence s∞ is ultimately periodic if there are non-negative


integers r, n0 with r ≥ 1 and n0 ≥ 0 such that sj+r = sj for all j ≥ n0 . The least
positive integer r with this property is called the period of s∞ and the
corresponding nonnegative integer n0 is called the preperiod of s∞ . The
sequence s∞ is periodic if its preperiod is 0.

Example 15: In this example, we work over F2 . Consider the sequence S∞


defined by
sj+4 = sj + sj+1 + sj+2 + sj+3 (17)
over F = F2 . In other words, we take c0 = c1 = c2 = c3 = 1. Construct a
sequence with initial state (1, 1, 1, 0). Also, find the preperiod. 69
Study Guide-II Solution: We have
s4 = s0 + s1 + s2 + s3 = 1 + 1 + 1 + 0 = 1
s5 = s1 + s2 + s3 + s4 = 1 + 1 + 0 + 1 = 1
s6 = s2 + s3 + s4 + s5 = 1 + 0 + 1 + 1 = 1
and so on. The sequence we get is
(1, 1, 1, 0, 1, 1, 1, 1, 0, 1, 1, 1, 1, 0, 1, 1, 1, 1, 0, 1, . . .) (18)
Note that, starting from the 2nd term s1 , the sequence repeats itself again and
again i.e.
sj+5 = sj ∀ j ≥ 1.
Also, 5 is the period and the preperiod is 1 in this case.
∗∗∗
Is this true in general, i.e. given any LFSR, does it repeat itself? If it repeats
always, can we predict after how many terms it will start repeating itself? We will
investigate these questions now.

The tool for our investigation is the shift operator S defined by


S(s0 , s1 , s2 , . . . , ) = (s1 , s2 , s3 , . . . , ) (19)
Also,
Sr s∞ = (sr , sr+1 , . . . , ) (20)
Check that, if s is the sequence in Eqn. (18), then,
Sj+5 s∞ = Sj s∞ ∀ j ≥ 1
More generally, we can formulate the periodicity property of a sequence in terms
of the shift operator. A sequence s∞ is periodic of period r if and only if
Sr s∞ = s∞ . This is because, we have
Sr (s0 , s1 , . . . , ) = (sr , sr+1 , . . .)
= (s0 , s1 , . . . , ) (∵ sn = sn+r for n = 0, 1, 2, . . . , )
Similarly, a sequence s∞ is ultimately periodic if and only if there are
non-negative integers r, n0 such that
Sj+r s∞ = Sj s∞
for j ≥ n0 . So, the question boils down to this: Given any sequence s∞ satisfying
Eqn. (16), can we find integers n0 ≥ 0, r > 0 such that Sj+r s∞ = Sr s∞ for j ≥ n0 ?

Consider

V = s∞ = (s0 , s1 , . . . , )| s0 , s1 , . . . ∈ Fq , s∞ satisfies Eqn. (16) (21)
Check that V is vector space over Fq under pointwise addition and scalar
multiplication. Further, S is a linear operator on V and it satisfies the equation
Sn − cn−1 Sn−1 − cn−2 Sn−2 − . . . − c1 S − c0 I = 0 (22)
Finite fields have the remarkable property that every nonzero element is a root of
unity. Using this we can see that if f(x) ∈ Fq [x] is such that f(0) =  0, then f(x)
divides x − 1 for some e ∈ N. In fact, as we show below, e can be so chosen that
e

e ≤ qm − 1, where m = deg(f(x)).
Lemma 1: Let f(x) ∈ Fq [x] be a polynomial of degree m ≥ 1 with f(0) = 0.
Then, there exists e ∈ N with e ≤ q − 1 such that f(x) divides x − 1.
m e

Proof: Consider the ring R = F[x]/f(x). Since f(0) =


 0 we cannot have
70 x = f(x)g(x) for any i ≥ 1 and g(x) ∈ Fq [x]. (Why?) Thus, the elements
i
xi + f(x) are nonzero in R for each i ≥ 1. Also, 1 + f(x) is nonzero in R since Applications of Finite Fields
m ≥ 1. Now R has qm elements and qm − 1 nonzero elements. Hence, by
pigeonhole principle, at least two of the nonzero elements xi + f(x),
i = 0, 1 . . . , qm − 1 must coincide. So, there are integers r and s, r < s, such that The Pigeonhole Principle
xr + f(x) = xs + f(x) says that, if more than n
pigeons are put in n holes,
and consequently f(x) divides xr (xe − 1), where e = s − r. But, f(0) = 0 so f(x) then at least one hole will
and xr are relatively prime in Fq . Thus, f(x) divides xe − 1 with contain two or more
1 ≤ e ≤ qm − 1.  pigeons.

Using Lemma 1 we can prove the following proprosition.


Proposition 3: For the sequence s∞ generated by the LFSR in Eqn. (16) of order
n over Fq , we have the following:
1) s∞ is ultimately periodic and its period is ≤ qn − 1.
2) If c0 = 0, then s∞ is periodic.

Proof: Let f(x) = xn − cn−1 xn−1 − . . . − c1 x − c0 . If f(0) = 0, then, by Lemma 1,


the polynomial f(x) has order ≤ qn − 1. In other words, f(x) | xm − 1 with
m ≤ qn − 1. Since S satisfies f(x) and f(x) divides xm − 1, S also satisfies the
polynomial f(x). In other words, Sm − I = 0 or Sm s∞ = s∞ , i.e. s∞ is periodic of
period m. If c0 = 0, f(0) = 0. So, in this case S is periodic of period ≤ qn − 1.

Suppose f(0) = 0. Then, c0 = 0. Let k be the smallest value for which ck = 0.


Then, f(x) = xk (xn−k − cn−k−1
x − . . . − ck ) = xk g(x) where g(0) = 0. So, period
of g(x) is ≤ qn−k − 1. By definition of period, this means that g(x) | xm − 1 with
m ≤ qn−k − 1. So, f(x) | xm+k − xk with m ≤ qn−k − 1. Also, Sm+j − Sj = 0 for
j ≥ k. So, Sm+j s∞ = Sj s∞ for j ≥ k. 

If the period of an LFSR is large, a large number of terms of the LFSR is non
repeating. So, we are interested in finding the period of an LFSR. We begin with
the following definiton.
Definition 7: An LFSR of order n over Fq is primitive if for any choice of a
nonzero initial state, the sequence generated by that LFSR is periodic of period
qn − 1.
Definition 8: Let f(x) ∈ Fq [x] be such that f(0) = 0. The smallest positive integer
e such that f(x) | xe − 1 is called the order of f(x). Suppose f(0) = 0 and h ∈ N
is such that xh | f(x) and xh+1  f(x). Writing f(x) = xh g(x), we define the order
of f(x) to be the order of g(x). We write ord(f(x)) or ord(f) for the order of f(x).

The terms exponent and period are also used in the literature instead of order.
Note that, if f(x) ∈ Fq [x] is irreducible of degree m then it has a root in Fqm .
Indeed, Fq [x]/f(x) is a field with qm elements and hence isomorphic to Fqm . In
m
fact, since Fqm is a normal extension of Fq (being the splitting field of xq − x
over Fq ), f(x) has all its roots in Fqm . We can relate the order of f(x) to the order
of its roots in F∗qm , provided f(0) = 0.
Proposition 4: Let f ∈ Fq [x] be irreducible of degree m and with f(0) = 0, and
let α ∈ Fqm be a root of f(x). Then, α = 0 and the order of f(x) is the order of α
in the multiplicative group F∗qm . 71
Study Guide-II Proof: Since f(0) = 0, we see that α = 0. Since f(x) is irreducible, it has the
least degree among all polynomials in Fq [x] having α as a root. So, if g(α) = 0
for any g(x) ∈ Fq [x], then f(x) | g(x). Thus for any d ≥ 1,
α d = 1 ⇔ α is a root of xd − 1 ⇔ f(x)|xd − 1
This implies that
{d ∈ N | f(x) divides xd − 1} = {d ∈ N|α d = 1}.
∴ min{d ∈ N | f(x) divides xd − 1} = min{d ∈ N|α d = 1}.
In other words, ord(f) is the order of α in F∗qm . 

Since Fqm is cyclic of order q − 1, it follows from Proposition 4 that there exist
m

irreducible polynomials in Fq [x] of the maximum possible order, viz., qm − 1.


(How?) These, when normalised to make the leading coefficient to be one, have a
special name.
Definition 9: A polynomial f(x) ∈ Fq [x] of degree m ∈ N is said to be a
primitive polynomial if it is monic, irreducible, and ord(f) = qm − 1.
Equivalently, f(x) ∈ Fq [x] of degree m is a primitive polynomial if it is the
minimal polynomial over Fq of a generator of the multiplicative cyclic group F∗qm .
Remark 5: Don’t confuse this definition of primitive polynomial with another
definition where a polynomial over a unique factorisation domain is called
primitive if the gcd of its coefficients is 1.

Note that, because of Proposition 4, to check whether an irreducible polynomial


f(x) ∈ Fq [x] of degree m is primitive, we have to select a root α of f(x) and check
whether it has order qm − 1. The following result from group theory is useful for
finding the order of an element in a group.
Lemma 2: Let G be a finite abelian group of order n and let g ∈ G. Suppose
n = pe11 pe22 · · · pekk , where p1 , p2 , . . ., pk are distinct primes e1 , e2 , . . ., ek ∈ N.
Write tj = pnej and gj = gtj for j = 1, . . ., k. Then, g has order n if and only if
ej −1
p
gj = 1 for j = 1, 2, . . . k.

We leave the proof of this lemma to you as an exercise. (See E 11).) Let us now
look at some examples.
Example 16: Check that, the polynomial f(x) = x4 + x + 1 ∈ F2 [x] is irreducible.
Find the order of the polynomial f(x).
Solution:
   = 0 and f(1)
Since f(0)  = 0, f(x) has no linear factors. Suppose
f(x) = x2 + ax + b x2 + cx + d , a, b, c, d ∈ F2 . We have
f(x) = x4 + (a + c)x3 + (ac + b + d)x2 + (ad + bc)x + bd = x4 + x + 1
So,
a+c = 0 (23)
ac + b + d = 0 (24)
ad + bc = 1 (25)
bd = 1 (26)
From Eqn. (26), it follows that b = d = 1. So, from Eqn. (24) it follows that
a = 0 or c = 0. But, from Eqn. (23) it follows that a = c, so a = c = 0. In this
case Eqn. (25) will not be satisfied, so f(x) is irreducible over F2 . We have
72 L  F2 [x]/f(x)
Let α = x + f(x). Then, α is a root of f(x) in F24  F2 [x]/f(x). Let us find Applications of Finite Fields
the order of α. We have |L∗ | = 15 = 3 · 5. Comparing with Lemma 2, here k = 2,
p1 = 3, p2 = 5, e1 = 1, e2 = 1. So, t1 = 153 = 5 and t2 = 5 = 3. Also,
15

g1 = α t1 = α 5 and g2 = α t2 = α 3 . To check that α has order 15, we have to


check that g1 = 1 and g2 = 1. We have
α 3 = x3 + f(x) = 1
α 4 = x4 + f(x) = −x − 1 + f(x) = x + 1 + f(x)(∵ −1 = 1 in F2 )
g1 = α 5 = (x + f(x)) (1 + x + f(x)) = x2 + x + f(x) = 1 + f(x)
g2 = α 3 = x3 + f(x) = 1
So, α has order 15, in other words, α generates |L∗ |. Therefore, the order of f is
15 and f is a primitive polynomial.
∗∗∗
Here are some exercises to test your understanding of the concepts of order and
primitive polynomials.

E9) If f(x) ∈ Fq [x] has degree m and order m, then show that f(0) = 0. EXERCISES

E10) Prove Lemma 2.

E11) Find the order of the following polynomials. Check whether they are
primitive:
i) x4 + x3 + x2 + x + 1 ∈ F2 [x] ii) x2 + x + 2 ∈ F3 [x]
iii) x3 + x2 + 1 ∈ F2 [x]

We have the following characterisation of primitive LFSRs.


Proposition 5: An LFSR of order n over Fq is primitive if and only if its
characteristic polynomial is primitive of degree n in Fq [x].

Let us now look at an example now.


Example 17: Let us take p = 2 and j = 5, c0 = c2 = 1, c1 = c3 = c4 = 0. Let
the linear recurrence relation be
sn+5 ≡ sn+2 + sn (mod 2). (27)
Check whether the LFSR satisfying Eqn. (27) is primitive.

Solution: The characteristic polynomial of the recurrence is x5 + x3 + 1 and


this is an irreducible polynomial over F2 (Check this.). Its root generates the finite
field F25 . Why is this so? Note that, the multiplicative group of F25 has 31
elements, i.e. it is of prime order. So, any non-zero element will generate the
multiplicative group. In particular, any root of the polynomial x5 + x3 + 1 will
generate the multiplicative group. So, the characteristic polynomial of Eqn. (27)
is primtive and so the LFSR is primitive by Proposition 5.
∗∗∗
Let us now see how we can use LFSRs for encryption and decryption.
Example 18: Consider the sequence
010000100101100111110001101110101. (28)
generated using Eqn. (27) and choosing (0, 1, 0, 0, 0) as the initial vector. Explain
how you will encrypt the text 1111000111010110 using this sequence. 73
Study Guide-II Solution: The plain text 1011001110001111 is of length 16. So, we choose
the first 16 terms of the sequence in Eqn. (28) as the key. We encrypt by adding
the terms the key with the terms of the plain text to get the following:
(plaintext) 1111000111010110
(key) + 0100001001011001
(ciphertext) 1011001110001111
Note that, the addtion is in F2 . We decrypt by adding the key sequence to the
ciphertext in exactly the same way.
(ciphertext) 1011001110001111
(key) + 0100001001011001
(plaintext) 1111000111010110
∗∗∗
Here are some exercises for you to check your understanding of the discussion so
far.

EXERCISES E12) Consider the following LFSR:


sn+2 = 4sn + 3sn+1
defined over F5 . Check whether it is primitive.
E13) Consider the LFSR
sn+4 = sn+1 + sn
defined over F2 . We know from Example 16 that this is a primitive
polynomial. Starting with the initial state (1, 0, 0, 1), generate a string of
length 10 and encrypt the text 1010110011.

We conclude this Unit by giving a summary of the Unit in the next section.

11.5 SUMMARY

In this Unit, we have discussed the following:


1) definition of a design and some examples of designs;
2) definition of an error correcting code and some examples of error correcting
codes;
3) definition of the generator matrix and the parity check matrix of an error
correcting code;
4) define a LFSR;
5) some application of LFSRs in cryptography;

11.6 SOLUTIONS/ANSWERS

E1) Here, ν = 7, k = 5. As a four design, we have t = 4. We have


λ = C(ν − t, ν − k) = C(7 − 4, 7 − 5) = C(3, 2) = 3.
When t = 3,
74 λ = C(ν − t, ν − k) = C(7 − 3, 7 − 5) = C(4, 2) = 6.
Applications of Finite Fields
E2) Let
{(J, B)|B ∈ B, J ⊂ X, J ∩ B = φ } . (29)
We can choose a set of size j from X in C(ν, j) ways. For each such set J,
there are λ (j) choices for B with B ∩ J = φ . So, the number of elements in
the set in Eqn. (29) is λ (j) C(ν, j).
On the other hand, if we fix a block B, check that we can find C(ν − k, j)
sets J ⊂ X such that J ∩ B = φ . Since there are b choices for the block B,
the number of elements in the set in Eqn. (29) is bC(ν − k, j). So, we get
λ (j) C(ν, j) = bC(ν − k, j) (30)
Using Eqn. (9), we have
C(ν, t)
λ (j) C(ν, j) = λ C(ν − k, j)
C(k, t)
or
C(ν, t)C(ν − k, j)
λ (j) = λ
C(k, t)C(ν, j)
ν! (ν−k)! (ν−j)!
(j) (ν−t)!t! (ν−k−j)!j! (ν−k−j)!k!
λ =λ k! ν!
=λ (ν−t)!
(k−t)!t! (ν−j)!j! (ν−k)!(k−t)!
C(ν − j, k)

C(ν − t, k − t)

E3) Putting t = 3, k = 6, λ = 1 and i = 2 in Eqn. (6), we get


C(v − 2, 1) ν − 2
λ2 = =
C(4, 1) 4
∴ v − 2 = 4λ2 or ν ≡ 2 (mod 4)
Check that, by putting t = 3, k = 6, λ = 1 and i = 1 in Eqn. (6), we get
(ν − 1)(ν − 2) = 20λ1 or (ν − 1)(ν − 2) ≡ 0 (mod 20) (31)
But, since ν ≡ 2 (mod 4), by considering the natural map Z20 → Z4 , you
can easily show that ν ≡ 2, 6, 10 or 14 (mod 20). Further, check that
Eqn. (31) is satisfied only for ν ≡ 2 or 6 (mod 20).

E4) The first set of parameters satisfy Eqn. (12) and Eqn. (13). So, there could
be a design with these parameters. Indeed, these are the parameters of the
Fano Plane. The second set of parameters do not satisfy Eqn. (13). So, there
can’t be a block design with these parameters.

E5) In this exercise, we will slightly deviate from our standard notation and
write codewords as tuples rather than strings. Consider the map from F21 to
F14 given by
(a1 , a2 , . . . , a21 ) → (a1 − a8 , a1 − a15 , a2 − a9 , a2 − a16 , . . . , a7 − a14 , . . . , a7 − a21 )
Check that the kernel of this map is precisely the code in Example 9.
Further, check that this map is surjective. Then, use rank-nullity theorem to
show that C is a seven dimensional subspace of F21 .
E6) For convenience, let us split the string we received into three chunks of
length seven and write them one below another and decode as in Example 9.
Only in the sixth column, there are 1s in the first two rows and there is a 0
in the third row. Since we assume that there is at most one error, the error 75
Study Guide-II must be in third row, sixth column and we decode the sixth bit as 1.
1 0 0 0 1 1 1
1 0 0 0 1 1 1
1 0 0 0 1 0 1
1 0 0 0 1 1 1
E7) Note that, all the rwo vectors satisfy the condition xi = xj if i ≡ j (mod 7),
so all the rows are valid codewords. Also, the generator matrix contains a
7 × 7 identity matrix, so it has rank 7.

E8) Note that, this is the generator matrix of the repetition codedefined  over F9 .
I
This is of the form [I3 , I3 , I3 ]. Here P = [I3 , I3 ]. So, −Pt = 3 = Q(say).
I3
So, the parity check matrix is
⎡ ⎤
1 0 0 1 0 0 0 0 0
⎢0 1 0 0 1 0 0 0 0⎥
⎢ ⎥
⎢0 0 1 0 0 1 0 0 0⎥
H = [Q, I6 ] = ⎢⎢ ⎥
1 0 0 0 0 0 1 0 0⎥
⎢ ⎥
⎣0 1 0 0 0 0 0 1 0⎦
0 0 1 0 0 0 0 0 1
E9) Suppose f(0) = 0. Then, f(x) = xi g(x) where i ≥ 1 and g(x) has degree
strictly greater than one and g(0) = 0. By the definition of order, we have
ord(f) = ord(g). Since the degree of g(x) is less than m and the order of
g(x) is at most the degree of g(x), if follows that ord(f) < m, a
contradiction.

E10) let O(g) denote the order of g ∈ G. In general, we have


  O(g)
O gk = . (32)
(k, O(g))
Suppose g has order n. Then, gn = 1. Use Eqn. (32) to show that gj = gtj
nj −1
n q
has order qj j . So, gj j = 1.
nj −1
q
Conversely, suppose that = 1 and gj j = 1 for each j. We have O(g) | n
gn
n  
since gn = 1. Further, gj has order qj j . Since O(gj ) = O gtj and
O(gk ) | O(g) for all k ∈ N, from Eqn. (32), it follows that qnj | O(g). Since
qn11 , qn22 , · · · are pairwise coprime, it follows that n | O(g). Since n | O(g)
and O(g) | n, it follows that O(g) = n.

E11) i) Let f(x) = x4 + x3 + x2 + x + 1. As before, we can check that the


polynomial has no linear factors. We write
  
f(x) = x2 + ax + b x2 + cx + d
As in Example 16, we have
a+c = 1 (33)
ac + b + d = 1 (34)
ad + bc = 1 (35)
bd = 1 (36)
From Eqn. (33), we get that only one of a or c can be 1. So, ac = 0.
76 From Eqn. (34), we get b + d = 1 which again implies that only of b
and d can be 1. So, bd = 0. But, this contradicts Eqn. (36). So, f(x) is Applications of Finite Fields
irreducible.
We also know that its roots lie in an extension of degree four over F2 .
If K is the splitting field of f(x), |K∗ | has 24 − 1 = 15 elements and the
order of any root of f(x) has to divide 15. If we represent K by
F[x]/hf(x)i and write α = x + hf(x)i, then α is a root of f(x) and its
order in K∗ will divide 15. We have
α 3 = x3 + hf(x)i 6= 1 + hf(x)i
Let us now find α 5 . We have
α 4 = x4 + hf(x)i
= x3 + x2 + x + 1 + hf(x)i
∴ α 5 = x4 + x3 + x2 + x + hf(x)i = 1 + hf(x)i
So, ord(f) = 5 6= 24 − 1 and thus f(x) is not a primitive polynomial.
ii) Let us write h(x) = x2 + x + 2. Let us first check irreducibility. We
have
h(0) 6= 0, h(1) 6= 0 and h(2) 6= 0
By factor theorem, h(x) does not have a root in F. If it splits into
factors of smaller degree, it will have two factors of degree one in F,
so, it will have a root in F. So, h(x) is irreducible over F3 . Its splitting
field L is an extension of degree two over F3 and so L∗ will have
32 − 1 = 8 elements. Let us represent L as F3 [x]/hh(x)i and consider
α = x + hh(x)i, a root of h(x) in L. We have to find the order of α in
L. We have
α 2 = x2 + hh(x)i = −x − 2 + hh(x)i = −x + 1 + hh(x)i
α 4 = (−x + 1)2 + hh(x)i = x2 − 2x + 1 + hh(x)i
= −x + 1 − 2x + 1 + hh(x)i = 2 + hh(x)i
∴ α 8 = 4 + hh(x)i = 1 + hh(x)i
So, α has order eight in L∗ . It follows that the order of h is eight and it
is a primitive polynomial.
iii) Let us write g(x) = x3 + x + 1. It is a simple matter to check that it has
no roots in F2 . We have g(0) 6= 0, and g(1) 6= 0. If it splits into factors
of smaller degrees, it will either split into two polynomials, one of
degree one and one of degree two or split into three linear factors. In
either case, it will have a root in F. So, g(x) is irreducible in F. So, the
roots of g(x) will lie in a splitting field L of degree three over F2 and
L∗ will have 23 − 1 = 7 elements. Since L∗ is of prime order, every
element other than 1 will have order 7. Since 1 is not a root of g(x),
every root of g(x) will have order 7. So g(x) has order 7 and it is a
primitive polynomial. In fact, every irreducible polynomial of degree 3
over F2 is a primitive polynomial because no irreducible polynomial of
degree 3 over F2 will have 1 as a root.

E12) The Characteristic polynomial of the recurrence is


f(x) = x2 − 4x − 3 = x2 + x + 2 since −4 ≡ 1 (mod 5) and −3 ≡ 2
(mod 5). We leave it to you to check that f(x) is irreducible. Let
L = F5 [x]/hf(x)i. Then, L is a finite field with 25 elements and
α = x + hf(x)i is root of f(x) in L. To check that f(x) is a primitive
polynomial, it is enough to check that α generates L∗ . For this, we have to 77
Study Guide-II check that the order of α is 24. Therefore, we need to check that
α 8 = 1 + f(x) and α 12 = 1 + f(x). We have
α 2 = x2 + f(x) = −x − 2 + f(x)
α 4 = (−x − 2)2 + f(x)
= x2 + 4x + 4 + f(x) = −x − 2 + 4x + 4 + f(x)
= 3x + 2 + f(x)
α 8 = (3x + 2)2 + f(x) = 9x2 + 12x + 4 + f(x)
= −x2 + 2x − 1 + f(x) = x + 2 + 2x − 1 + f(x)
= 3x + 1 + f(x) = 1 + f(x)
α 12 = α 8 α 4 = (3x + 1)(3x + 2) + f(x)
= 9x2 + 9x + 2 = −x2 − x + 2 + f(x) = 4 + f(x) = 1 + f(x)

E13) We have
s 4 = s1 + s0 = 1 s5 = s2 + s1 = 0 s6 = s3 + s2 = 1
s7 = s4 + s3 = 0 s8 = s5 + s4 = 1 s9 = s6 + s5 = 1
s10 = s7 + s6 = 1
We use the string we generated to encrypt the given text as follows:
(plaintext) 1010110011
(key) + 1001010111
(ciphertext) 0011100100

78
FURTHER READING Applications of Finite Fields

[1] Jürgen Bierbrauer. (2000) Introduction to group theory and applications,


lecture notes. available from
http://www.math.mtu.edu/~jbierbra/HOMEZEUGS/groupssalz.ps.

[2] S. R. Ghorpade, S. U. Hasan, and M. Kumari, “Primitive polynomials, singer


cycles and word-oriented linear feedback shift registers,” Designs, Codes and
Cryptography, vol. 58, no. 2, pp. 123–134, 2011.

[3] K. Joshi, Applied Discrete Structures. New Age International, 1997.

[4] R. Lidl and H. Niederreiter, Finite Fields, ser. Encyclopedia of Mathematics


and its Applications. Cambridge University Press, 1996, no. v. 20, pt. 1.

[5] J. v. Lint and R. Wilson, A Course in Combinatorics, Second ed.


Cambridge University Press, 2001.

[6] F. J. Macwilliams and N. J. A. Sloane, The Theory of Error-correcting Codes.


North Holland: Elsevier Science Publishers, 1977.

79

You might also like